MED SURG EXAM 2 CH 50-54

Réussis tes devoirs et examens dès maintenant avec Quizwiz!

A 54-year-old patient is on the surgical unit after a radical abdominal hysterectomy. Which finding is most important to report to the health care provider? a. Urine output of 125 mL in the first 8 hours after surgery b. Decreased bowel sounds in all four abdominal quadrants c. One-inch area of bloody drainage on the abdominal dressing d. Complaints of abdominal pain at the incision site with coughing

A The decreased urine output indicates possible low blood volume and further assessment is needed to assess for possible internal bleeding. Decreased bowel sounds, minor drainage on the dressing, and abdominal pain with coughing are expected after this surgery. DIF: Cognitive Level: Apply (application) REF: 1296 TOP: Nursing Process: Assessment MSC:

A pt. who had breast cancer is now having breast reconstructive surgery. To enhance the quality of the reconstructive surgery, what is the nurse's priority patient assessment when the patient returns for enlargement of the expander? A. Color of the affected skin B. Attitiude on reconstruction C. Desired size of new breast D. Tenderness near expander

A. Color of the affected skin. The nurses priority assessment is the color of the skin over and surrounding the expander because abnormal skin color can indicate poor perfusion to the region and risk the viability to the tissue. This is the nurses priority because the purpose of teh expander is to stretch the patiesn skin that will eventually cover the breast implant. The skin over the expander should be pink, warm, and supple. Skin with poor perfusion can look dusky and pale and be cool and friable.

The nurse is teaching a group of young women about prevention of sexually transmitted infections (STIs). Which method of contraception also helps give some protection against STIs? A. Condoms B. Intrauterine devices C. Oral contraceptive pills D. Medroxyprogesterone

A. Condoms

When performing an assessment of a patient with anorectal gonorrhea, what are the symptoms that the nurse is likely to find? Select all that apply. A. Mucopurulent anal discharge B. Bleeding C. Tenesmus D. Loose watery stools E. Hemorrhoids

A. Mucopurulent anal discharge B. Bleeding C. Tenesmus

While interviewing a patient, the nurse finds that the patient uses superabsorbent tampons during the menstrual cycle. For which symptoms should the nurse tell the patient to be observant? Select all that apply. A. Myalgia B. Vomiting C. Diarrhea D. Low fever E. Dizziness

A. Myalgia B. Vomiting C. Diarrhea

A patient has been diagnosed with a few small leiomyomas, and she has informed the health care provider that she would like to preserve her uterus. Regarding which procedure should the nurse prepare to educate the patient? A. Myomectomy B. Hysteroscopy C. Dilation and curettage D. Laparoscopic hysterectomy

A. Myomectomy

When teaching a group of young women about pelvic inflammatory disease (PID), what information should the nurse include? Select all that apply. A. PID is associated with a higher risk of ectopic pregnancy. B. Old age and multiple sex partners are risk factors for PID. C. Causative organisms reach the pelvic organs through blood. D. PID is an infectious condition of the fallopian tubes, ovaries, and uterus. E. Causative organisms reach the pelvic organs through the cervix in an ascending manner.

A. PID is associated with a higher risk of ectopic pregnancy. E. Causative organisms reach the pelvic organs through the cervix in an ascending manner.

The nurse is educating a patient about decreasing the risk of developing deep vein thrombosis (DVT). What should the nurse be sure to include when discussing this with the patient? Select all that apply A. Perform leg exercises B. Change position frequently C. Maintain high-Fowler's position D. Avoid pressure under the knees E. Massage legs when they get stiff in bed

A. Perform leg exercises B. Change position frequently D. Avoid pressure under the knees

A patient is suspected to have syphilis. Which stage of syphilis is most infectious and may have chancres? A. Primary B. Secondary C. Latent D. Tertiary

A. Primary

A couple married for two years has failed to conceive. What advice should the nurse give to the couple? Select all that apply. A. Provide information regarding physiology of reproduction and explain various factors which result in infertility. B. Instruct the couple that because they have failed to conceive within two years after their marriage, it will be difficult for them to conceive. C. Inform the couple that infertility management is very expensive and ask them to obtain insurance coverage before they begin routine testing. D. Inform the couple they should directly go for assisted reproductive technologies such as in vitro fertilization and zygote intrafallopian transfer. E. Encourage the couple to identify the psychological problems, emotional disturbances, social distress, and frustration and anger associated with infertility and to address these issues accordingly.

A. Provide information regarding physiology of reproduction and explain various factors which result in infertility. E. Encourage the couple to identify the psychological problems, emotional disturbances, social distress, and frustration and anger associated with infertility and to address these issues accordingly.

When performing discharge teaching for a patient after a vasectomy, the nurse instructs the patient that he a. should continue to use other methods of birth control for 6 weeks. b. should not have sexual intercourse until his 6-week follow-up visit. c. may have temporary erectile dysfunction (ED) because of swelling. d. will notice a decrease in the appearance and volume of his ejaculate.

ANS: A Because it takes about 6 weeks to evacuate sperm that are distal to the vasectomy site, the patient should use contraception for 6 weeks. ED that occurs after vasectomy is psychologic in origin and not related to postoperative swelling. The patient does not need to abstain from intercourse. The appearance and volume of the ejaculate are not changed because sperm are a minor component of the ejaculate

Which infection, reported in the health history of a woman who is having difficulty conceiving, will the nurse identify as a risk factor for infertility? a. N. gonorrhoeae b. Treponema pallidum c. Condyloma acuminatum d. Herpes simplex virus type 2

ANS: A Complications of gonorrhea include scarring of the fallopian tubes, which can lead to tubal pregnancies and infertility. Syphilis, genital warts, and genital herpes do not lead to problems with conceiving, although transmission to the fetus (syphilis) or newborn (genital warts or genital herpes) is a concern

Which patient will the nurse plan on teaching about the Gardasil vaccine? a. A 24-year-old female who has not been sexually active b. A 34-year-old woman who has multiple sexual partners c. A 19-year-old woman who is pregnant for the first time d. A 29-year-old woman who is in a monogamous relationship

ANS: A Gardasil is recommended for females ages 9 through 26, preferably those who have never been sexually active. It is not recommended for women during pregnancy or for older women

The nurse will inform a patient with cancer of the prostate that side effects of leuprolide (Lupron) may include a. flushing. b. dizziness. c. infection. d. incontinence.

ANS: A Hot flashes may occur with decreased testosterone production. Dizziness may occur with the alpha-blockers used for benign prostatic hyperplasia (BPH). Urinary incontinence may occur after prostate surgery, but it is not an expected side effect of medication. Risk for infection is increased in patients receiving chemotherapy

A 46-year-old man who has had blood drawn for an insurance screening has a positive Venereal Disease Research Laboratory (VDRL) test. Which action should the nurse take next? a. Ask the patient about past treatment for syphilis. b. Explain the need for blood and spinal fluid cultures. c. Obtain a specimen for fluorescent treponemal antibody absorption (FAT-Abs) testing. d. Assess for the presence of chancres, flulike symptoms, or a bilateral rash on the trunk.

ANS: A Once antibody testing is positive for syphilis, the antibodies remain present for an indefinite period of time even after successful treatment, so the nurse should inquire about previous treatment before doing other assessments or testing. Culture, FAT-Abs testing, and assessment for symptoms may be appropriate, based on whether the patient has been previously treated for syphilis

A 29-year-old female patient is diagnosed with Chlamydia during a routine pelvic examination. The nurse knows that teaching regarding the management of the condition has been effective when the patient says which of the following? a. "My partner will need to take antibiotics at the same time I do." b. "Go ahead and give me the antibiotic injection, so I will be cured." c. "I will use condoms during sex until I finish taking all the antibiotics." d. "I do not plan on having children, so treating the infection is not important."

ANS: A Sex partners should be treated simultaneously to prevent reinfection. Chlamydia is treated with oral antibiotics. Abstinence from sexual intercourse is recommended for 7 days after treatment, and condoms should be recommended during all sexual contacts to prevent infection. Chronic pelvic pain, as well as infertility, can result from untreated Chlamydia

A 20-year-old woman who is being seen in the family medicine clinic for an annual physical exam reports being sexually active. The nurse will plan to teach the patient about a. testing for Chlamydia infection. b. immunization for herpes simplex. c. infertility associated with the human papillomavirus (HPV). d. the relationship between the herpes virus and cervical cancer.

ANS: A Testing for Chlamydia is recommended for all sexually active females under age 25 by the Centers for Disease Control and Prevention. HPV infection does not cause infertility. There is no vaccine available for herpes simplex, and herpes simplex infection does not cause cervical cancer

Several patients call the urology clinic requesting appointments with the health care provider as soon as possible. Which patient will the nurse schedule to be seen first? a. 22-year-old who has noticed a firm, nontender lump on his scrotum b. 35-year-old who is concerned that his scrotum "feels like a bag of worms" c. 40-year-old who has pelvic pain while being treated for chronic prostatitis d. 70-year-old who is reporting frequent urinary dribbling after a prostatectomy

ANS: A The patient's age and symptoms suggest possible testicular cancer. Some forms of testicular cancer can be very aggressive, so the patient should be evaluated by the health care provider as soon as possible. Varicoceles do require treatment, but not emergently. Ongoing pelvic pain is common with chronic prostatitis. Urinary dribbling is a common problem after prostatectomy

The following male patients recently arrived in the emergency department. Which one should the nurse assess first? a. 19-year-old who is complaining of severe scrotal pain b. 60-year-old with a nontender ulceration of the glans penis c. 22-year-old who has purulent urethral drainage and back pain d. 64-year-old who has dysuria after brachytherapy for prostate cancer

ANS: A The patient's age and symptoms suggest possible testicular torsion, which will require rapid treatment in order to prevent testicular necrosis. The other patients also require assessment by the nurse, but their history and symptoms indicate nonemergent problems (acute prostatitis, cancer of the penis, and radiation-associated urinary tract irritation)

A 19-year-old patient has genital warts around her external genitalia and perianal area. She tells the nurse that she has not sought treatment until now because "the warts are so disgusting." Which nursing diagnosis is most appropriate? a. Disturbed body image related to feelings about the genital warts b. Ineffective coping related to denial of increased risk for infection c. Risk for infection related to lack of knowledge about transmission d. Anxiety related to impact of condition on interpersonal relationships

ANS: A The patient's statement that her lesions are disgusting suggests that disturbed body image is the major concern. There is no evidence to indicate ineffective coping or lack of knowledge about mode of transmission. The patient may be experiencing anxiety, but there is nothing in the data indicating that the genital warts are impacting interpersonal relationships.

Which information will the nurse plan to include when teaching a community health group about testicular self-examination? a. Testicular self-examination should be done in a warm room. b. The only structure normally felt in the scrotal sac is the testis. c. Testicular self-examination should be done at least every week. d. Call the health care provider if one testis is larger than the other.

ANS: A The testes will hang lower in the scrotum when the temperature is warm (e.g., during a shower), and it will be easier to palpate. The epididymis is also normally palpable in the scrotum. One testis is normally larger. The patient should perform testicular self-examination monthly

The nurse in the outpatient clinic notes that the following patients have not received the human papillomavirus (HPV) vaccine. Which patients should the nurse plan to teach about benefits of the vaccine (select all that apply)? a. 24-year-old man who has a history of genital warts b. 18-year-old man who has had one male sexual partner c. 28-year-old woman who has never been sexually active d. 20-year-old woman who has a newly diagnosed Chlamydia infection e. 30-year-old woman whose sexual partner has a history of genital warts

ANS: A, B, D The HPV vaccines are recommended for male and female patients between ages 9 through 26. Ideally, the vaccines are administered before patients are sexually active, but they offer benefit even to those who already have HPV infection

A 39-year-old patient with a history of IV drug use is seen at a community clinic. The patient reports difficulty walking, stating "I don't know where my feet are." Diagnostic screening reveals positive Venereal Disease Research Laboratory (VDRL) and fluorescent treponemal antibody absorption (FTA-Abs) tests. Based on the patient history, what will the nurse assess (select all that apply)? a. Heart sounds b. Genitalia for lesions c. Joints for swelling and inflammation d. Mental state for judgment and orientation e. Skin and mucous membranes for gummas

ANS: A, D, E The patient's clinical manifestations and laboratory tests are consistent with tertiary syphilis. Valvular insufficiency, gummas, and changes in mentation are other clinical manifestations of this stage

A 22-year-old man tells the nurse at the health clinic that he has recently had some problems with erectile dysfunction. Which question should the nurse ask first to assess for possible etiologic factors? a. "Do you experience an unusual amount of stress?" b. "Do you use any recreational drugs or drink alcohol?" c. "Do you have chronic cardiovascular or peripheral vascular disease?" d. "Do you have a history of an erection that lasted for 6 hours or more?"

ANS: B A common etiologic factor for erectile dysfunction (ED) in younger men is use of recreational drugs or alcohol. Stress, priapism, and cardiovascular illness also contribute to ED, but they are not common etiologic factors in younger men

The health care provider prescribes finasteride (Proscar) for a 67-year-old patient who has benign prostatic hyperplasia (BPH). When teaching the patient about the drug, the nurse informs him that a. he should change position from lying to standing slowly to avoid dizziness. b. his interest in sexual activity may decrease while he is taking the medication. c. improvement in the obstructive symptoms should occur within about 2 weeks. d. he will need to monitor his blood pressure frequently to assess for hypertension.

ANS: B A decrease in libido is a side effect of finasteride because of the androgen suppression that occurs with the drug. Although orthostatic hypotension may occur if the patient is also taking a medication for erectile dysfunction (ED), it should not occur with finasteride alone. Improvement in symptoms of obstruction takes about 6 months. The medication does not cause hypertension

A 22-year-old patient with gonorrhea is treated with a single IM dose of ceftriaxone (Rocephin) and is given a prescription for doxycycline (Vibramycin) 100 mg bid for 7 days. The nurse explains to the patient that this combination of antibiotics is prescribed to a. prevent reinfection during treatment. b. treat any coexisting chlamydial infection. c. eradicate resistant strains of N. gonorrhoeae. d. prevent the development of resistant organisms.

ANS: B Because there is a high incidence of co-infection with gonorrhea and chlamydia, patients are usually treated for both. The other explanations about the purpose of the antibiotic combination are not accurate

A 57-year-old patient is incontinent of urine following a radical retropubic prostatectomy. The nurse will plan to teach the patient a. to restrict oral fluid intake. b. pelvic floor muscle exercises. c. to perform intermittent self-catheterization. d. the use of belladonna and opium suppositories.

ANS: B Pelvic floor muscle training (Kegel) exercises are recommended to strengthen the pelvic floor muscles and improve urinary control. Belladonna and opium suppositories are used to reduce bladder spasms after surgery. Intermittent self-catheterization may be taught before surgery if the patient has urinary retention, but it will not be useful in reducing incontinence after surgery. The patient should have a daily oral intake of 2 to 3 L.

The nurse in a health clinic receives requests for appointments from several patients. Which patient should be seen by the health care provider first? a. A 48-year-old man who has perineal pain and a temperature of 100.4° F b. A 58-year-old man who has a painful erection that has lasted over 6 hours c. A 38-year-old man who states he had difficulty maintaining an erection last night d. A 68-year-old man who has pink urine after a transurethral resection of the prostate (TURP) 3 days ago

ANS: B Priapism can cause complications such as necrosis or hydronephrosis, and this patient should be treated immediately. The other patients do not require immediate action to prevent serious complications

The nurse in the clinic notes elevated prostate specific antigen (PSA) levels in the laboratory results of these patients. Which patient's PSA result is most important to report to the health care provider? a. A 38-year-old who is being treated for acute prostatitis b. A 48-year-old whose father died of metastatic prostate cancer c. A 52-year-old who goes on long bicycle rides every weekend d. A 75-year-old who uses saw palmetto to treat benign prostatic hyperplasia (BPH)

ANS: B The family history of prostate cancer and elevation of PSA indicate that further evaluation of the patient for prostate cancer is needed. Elevations in PSA for the other patients are not unusual

A 71-year-old patient who has benign prostatic hyperplasia (BPH) with urinary retention is admitted to the hospital with elevated blood urea nitrogen (BUN) and creatinine. Which prescribed therapy should the nurse implement first? a. Infuse normal saline at 50 mL/hr. b. Insert a urinary retention catheter. c. Draw blood for a complete blood count. d. Schedule a pelvic computed tomography (CT) scan.

ANS: B The patient data indicate that the patient may have acute kidney injury caused by the BPH. The initial therapy will be to insert a catheter. The other actions are also appropriate, but they can be implemented after the acute urinary retention is resolved

A 32-year-old woman who is diagnosed with Chlamydia tells the nurse that she is very angry because her husband is her only sexual partner. Which response should the nurse make first? a. "You may need professional counseling to help resolve your anger." b. "It is understandable that you are angry with your husband right now." c. "Your feelings are justified and you should share them with your husband." d. "It is important that both you and your husband be treated for the infection."

ANS: B This response expresses the nurse's acceptance of the patient's feelings and encourages further discussion and problem solving. The patient may need professional counseling, but more assessment of the patient is needed before making this judgment. The nurse should also assess further before suggesting that the patient share her feelings with the husband because problems such as abuse might be present in the relationship. Although it is important that both partners be treated, the patient's current anger suggests that this is not the appropriate time to bring this up

When caring for a patient with continuous bladder irrigation after having transurethral resection of the prostate, which action could the nurse delegate to unlicensed assistive personnel (UAP)? a. Teach the patient how to perform Kegel exercises. b. Report any complaints of pain or spasms to the nurse. c. Monitor for increases in bleeding or presence of clots. d. Increase the flow rate of the irrigation if clots are noted.

ANS: B UAP education and role includes reporting patient concerns to supervising nurses. Patient teaching, assessments for complications, and actions such as bladder irrigation require more education and should be done by licensed nursing staff

When a 31-year-old male patient returns to the clinic for follow-up after treatment for gonococcal urethritis, a purulent urethral discharge is still present. When trying to determine the reason for the recurrent infection, which question is most appropriate for the nurse to ask the patient? a. "Did you take the prescribed antibiotic for a week?" b. "Did you drink at least 2 quarts of fluids every day?" c. "Were your sexual partners treated with antibiotics?" d. "Do you wash your hands after using the bathroom?"

ANS: C A common reason for recurrence of symptoms is reinfection because infected partners have not been simultaneously treated. Because gonorrhea is treated with one dose of antibiotic, antibiotic therapy for a week is not needed. An adequate fluid intake is important, but a low fluid intake is not a likely cause for failed treatment. Poor hygiene may cause complications such as ocular trachoma but will not cause a failure of treatment

A woman is diagnosed with primary syphilis during her eighth week of pregnancy. The nurse will plan to teach the patient about the a. likelihood of a stillbirth. b. plans for cesarean section c. intramuscular injection of penicillin. d. antibiotic eye drops for the newborn.

ANS: C A single injection of penicillin is recommended to treat primary syphilis. This will treat the mother and prevent transmission of the disease to the fetus. Instillation of erythromycin into the eyes of the newborn is used to prevent gonorrheal eye infections. C-section is used to prevent the transmission of herpes to the newborn. Although stillbirth can occur if the fetus is infected with syphilis, treatment before the tenth week of gestation will eliminate in utero transmission to the fetus.

The health care provider prescribes the following interventions for a patient with acute prostatitis caused by E. coli. Which intervention should the nurse question? a. Instruct patient to avoid sexual intercourse until treatment is complete. b. Administer ibuprofen (Advil) 400 mg every 8 hours as needed for pain. c. Catheterize the patient as needed if symptoms of urinary retention develop. d. Give trimethoprim/sulfamethoxazole (Bactrim) DS 1 tablet daily for 28 days.

ANS: C Although acute urinary retention may occur, insertion of a catheter through an inflamed urethra is contraindicated and the nurse will anticipate that the health care provider will need to insert a suprapubic catheter. The other actions are appropriate

A 53-year-old man is scheduled for an annual physical exam. The nurse will plan to teach the patient about the purpose of a. urinalysis collection. b. uroflowmetry studies. c. prostate specific antigen (PSA) testing. d. transrectal ultrasound scanning (TRUS).

ANS: C An annual digital rectal exam (DRE) and PSA are usually recommended starting at age 50 for men who have an average risk for prostate cancer. Urinalysis and uroflowmetry studies are done if patients have symptoms of urinary tract infection or changes in the urinary stream. TRUS may be ordered if the DRE or PSA is abnormal

Which finding from the nurse's physical assessment of a 42-year-old male patient should be reported to the health care provider? a. One testis hangs lower than the other. b. Genital hair distribution is diamond shaped. c. Clear discharge is present at the penile meatus. d. Inguinal lymph nodes are nonpalpable bilaterally.

ANS: C Clear penile discharge may be indicative of a sexually transmitted infection (STI). The other findings are normal and do not need to be reported

The nurse taking a focused health history for a patient with possible testicular cancer will ask the patient about a history of a. testicular torsion. b. testicular trauma. c. undescended testicles. d. sexually transmitted infection (STI).

ANS: C Cryptorchidism is a risk factor for testicular cancer if it is not corrected before puberty. STI, testicular torsion, and testicular trauma are risk factors for other testicular conditions but not for testicular cancer

Which question should the nurse ask when assessing a 60-year-old patient who has a history of benign prostatic hyperplasia (BPH)? a. "Have you noticed any unusual discharge from your penis?" b. "Has there been any change in your sex life in the last year?" c. "Has there been a decrease in the force of your urinary stream?" d. "Have you been experiencing any difficulty in achieving an erection?"

ANS: C Enlargement of the prostate blocks the urethra, leading to urinary changes such as a decrease in the force of the urinary stream. The other questions address possible problems with infection or sexual difficulties, but they would not be helpful in determining whether there were functional changes caused by BPH.

A patient returning from surgery for a perineal radical prostatectomy will have a nursing diagnosis of risk for infection related to a. urinary incontinence. b. prolonged urinary stasis. c. possible fecal wound contamination. d. placement of a suprapubic bladder catheter.

ANS: C The perineal approach increases the risk for infection because the incision is located close to the anus and contamination with feces is possible. Urinary stasis and incontinence do not occur because the patient has a retention catheter in place for 1 to 2 weeks. A urethral catheter is used after the surgery

A 44-year-old patient in the sexually transmitted infection clinic has a positive Venereal Disease Research Laboratory (VDRL) test, but no chancre is visible on assessment. The nurse will plan to send specimens for a. gram stain. b. cytologic studies. c. rapid plasma reagin (RPR) agglutination. d. fluorescent treponemal antibody absorption (FTA-Abs).

ANS: D Because false positives are common with VDRL and RPR testing, FTA-Abs testing is recommended to confirm a diagnosis of syphilis. Gram staining is used for other sexually transmitted infections (STIs) such as gonorrhea and Chlamydia and cytologic studies are used to detect abnormal cells (such as neoplastic cells).

The nurse explains to a 37-year-old patient being prepared for colposcopy with a cervical biopsy that the procedure a. involves dilation of the cervix and biopsy of the tissue lining the uterus. b. will take place in a same-day surgery center so that local anesthesia can be used. c. requires that the patient have nothing to eat or drink for 6 hours before the procedure. d. is similar to a speculum examination of the cervix and should result in little discomfort.

ANS: D Colposcopy involves visualization of the cervix with a binocular microscope and is similar to a speculum examination. Anesthesia is not required and fasting is not necessary. A cervical biopsy may cause a minimal amount of pain.

The nurse will anticipate that a 61-year-old patient who has an enlarged prostate detected by digital rectal examination (DRE) and an elevated prostate specific antigen (PSA) level will need teaching about a. cystourethroscopy. b. uroflowmetry studies. c. magnetic resonance imaging (MRI). d. transrectal ultrasonography (TRUS).

ANS: D In a patient with an abnormal DRE and elevated PSA, transrectal ultrasound is used to visualize the prostate for biopsy. Uroflowmetry studies help determine the extent of urine blockage and treatment, but there is no indication that this is a problem for this patient. Cystoscopy may be used before prostatectomy but will not be done until after the TRUS and biopsy. MRI is used to determine whether prostatic cancer has metastasized but would not be ordered at this stage of the diagnostic process

A 32-year-old man who has a profuse, purulent urethral discharge with painful urination is seen at the clinic. Which information will be most important for the nurse to obtain? a. Contraceptive use b. Sexual orientation c. Immunization history d. Recent sexual contacts

ANS: D Information about sexual contacts is needed to help establish whether the patient has been exposed to a sexually transmitted infection (STI) and because sexual contacts also will need treatment. The other information also may be gathered but is not as important in determining the plan of care for the patient's current symptoms

The nurse will plan to teach a 51-year-old man who is scheduled for an annual physical exam about a(n) a. increased risk for testicular cancer. b. possible changes in erectile function. c. normal decreases in testosterone level. d. prostate specific antigen (PSA) testing.

ANS: D PSA testing may be recommended annually for men, starting at age 50. There is no indication that the other patient teaching topics are appropriate for this patient

A 76-year-old patient who has been diagnosed with stage 2 prostate cancer chooses the option of active surveillance. The nurse will plan to a. vaccinate the patient with sipuleucel-T ( Provenge). b. provide the patient with information about cryotherapy. c. teach the patient about placement of intraurethral stents. d. schedule the patient for annual prostate-specific antigen testing.

ANS: D Patients who opt for active surveillance need to have annual digital rectal exams and prostate-specific antigen testing. Vaccination with sipuleucel-T, cryotherapy, and stent placement are options for patients who choose to have active treatment for prostate cancer

After a transurethral resection of the prostate (TURP), a 64-year-old patient with continuous bladder irrigation complains of painful bladder spasms. The nurse observes clots in the urine. Which action should the nurse take first? a. Increase the flow rate of the bladder irrigation. b. Administer the prescribed IV morphine sulfate. c. Give the patient the prescribed belladonna and opium suppository. d. Manually instill and then withdraw 50 mL of saline into the catheter.

ANS: D The assessment suggests that obstruction by a clot is causing the bladder spasms, and the nurse's first action should be to irrigate the catheter manually and to try to remove the clots. IV morphine will not decrease the spasm, although pain may be reduced. Increasing the flow rate of the irrigation will further distend the bladder and may increase spasms. The belladonna and opium suppository will decrease bladder spasms but will not remove the obstructing blood clot

A woman calls the clinic because she is having an unusually heavy menstrual flow. She tells the nurse that she has saturated three tampons in the past 2 hours. The nurse estimates that the amount of blood loss over the past 2 hours is _____ mL. a. 20 to 30 b. 30 to 40 c. 40 to 60 d. 60 to 90

ANS: D The average tampon absorbs 20 to 30 mL.

A patient is scheduled to have a myomectomy. What statement made by the patient indicates to the nurse that the patient understands the procedure? A. "I am having an ovary removed" B. "I am having uterine fibroids removed" C. "I am having a fallopian tube removed" D. "I am having a portion of my vagina removed"

B. "I am having uterine fibroids removed"

A patient with cervical cancer asks the nurse about various treatment modalities. What information should the nurse provide to this patient? Select all that apply. A. Hormone-based chemotherapy regimens B. External radiotherapy or internal radiotherapy or both C. Surgery or radiation, or chemotherapy as single treatments based on patient's choice D. Management depends on stage of the tumor, patient's age, and general state of health E. Surgical procedures include hysterectomy and radical hysterectomy (involving adjacent structures)

B. External radiotherapy or internal radiotherapy or both D. Management depends on stage of the tumor, patient's age, and general state of health E. Surgical procedures include hysterectomy and radical hysterectomy (involving adjacent structures)

The nurse is performing an educational session for a group of women regarding cervical cancer. Which risk factors for cervical cancer should the nurse include in the presentation? Select all that apply. A. Infertility B. Smoking C. Dysmenorrhea D. Caucasian race E. Chlamydia infection F. Low socioeconomic class

B. Smoking E. Chlamydia infection F. Low socioeconomic class

A patient has a vesicovaginal fistula and undergoes colporrhaphy to prevent further complications. What instruction should the nurse provide to the patient after the procedure? A. "You should avoid taking stool softeners." B. "You should reduce fluid intake throughout the day." C. "You should avoid taking a warm sitz bath frequently." D. "You should ensure that a catheter is in place for 10 days."

D. "You should ensure that a catheter is in place for 10 days."

The nurse is obtaining a history and performing an assessment of a patient with polycystic ovarian syndrome (PCOS). Which clinical manifestations does the nurse anticipate observing in this patient? Select all that apply. A. Obesity B. Hirsutism C. Amenorrhea D. Hypertension E. Endometriosis F. Irregular menstrual periods

A. Obesity B. Hirsutism C. Amenorrhea F. Irregular menstrual periods

A patient with premenstrual syndrome is on medication therapy. On a follow up visit, the nurse observes that the patient's cholesterol level is 200 mg/dL, and her blood pressure is 140/90 mm Hg. Which medication does the nurse report may be responsible for this condition? A. Estrogen B. Methotrexate C. Tranexamic acid D. Medroxyprogesterone

A. Estrogen

A nurse is assessing a patient with a suspected sexually transmitted infection (STI). What questions about lifestyle patterns should a nurse include to assess high-risk behavior? Select all that apply. A. Illicit use of intravenous drugs B. Method of contraception used C. Sexual preference D. Diet and exercise regimen E. Number of sexual partners F. Smoking and alcohol consumption

A. Illicit use of intravenous drugs B. Method of contraception used C. Sexual preference E. Number of sexual partners

A postmenopausal woman is on combined hormone therapy with estrogen and progesterone. What information should the nurse provide to this patient regarding advantages of hormonal therapy? Select all that apply. A. It reduces the risk of hip fractures and colorectal cancer. B. The therapy is very beneficial in women without a uterus. C. The therapy decreases the risk of stroke and heart disease. D. It may cause breast enlargement, which is a common side effect. E. The therapy helps to treat hot flashes and vaginal atrophic changes.

A. It reduces the risk of hip fractures and colorectal cancer. D. It may cause breast enlargement, which is a common side effect. E. The therapy helps to treat hot flashes and vaginal atrophic changes.

A patient with polycystic ovary syndrome (PCOS) is prescribed metformin. The patient seeks an explanation, as the drug is usually prescribed for diabetes. What explanation should the nurse provide to this patient? Select all that apply. A. It restores ovulation. B. It reduces ovarian cysts. C. It reduces plasma insulin. D. It increases plasma insulin. E. It improves hyperandrogenism.

A. It restores ovulation. C. It reduces plasma insulin. E. It improves hyperandrogenism.

A pregnant patient reports occasional spotting and severe pain in the abdomen. The ultrasonography reports reveal the absence of an intrauterine fetus and serum β-hCGof 1,500 mIU per mL. For which procedure that may prevent complications does the nurse prepare the patient? A. Laparoscopy B. Myomectomy C. Hysterectomy D. Balloon thermotherapy

A. Laparoscopy

A patient is infected with the herpes virus exhibiting lesions on the genital area. What instructions should the nurse give to this patient? Select all that apply. A. Maintain good hygiene. B. Keep the lesions clean and dry. C. Take frequent sitz baths. D. Burst the vesicles to drain the fluid. E. Use hot fomentation on the vesicles. F. Wear loose-fitting cotton undergarments.

A. Maintain good hygiene. B. Keep the lesions clean and dry. C. Take frequent sitz baths. F. Wear loose-fitting cotton undergarments.

The normal physiologic cessation of menses associated with declining ovarian function is referred to as which condition? A. Menopause B. Amenorrhea C. Dysmenorrhea D. Perimenopause

A. Menopause

The nurse is caring for a patient with an ectopic pregnancy of more than 3 cm in size. What nursing interventions should the nurse perform for this patient? Select all that apply. A. Monitor vital signs around the clock to identify signs of shock. B. Prepare the patient for either laparoscopic or laparotomy surgery. C. Assure the patient that with proper care she may continue the pregnancy. D. Inform the patient that laparotomies are associated with fewer repeated ectopic pregnancies. E. Inform the patient that increasing pain and vaginal bleeding indicate dislodgement of the ectopic pregnancy.

A. Monitor vital signs around the clock to identify signs of shock. B. Prepare the patient for either laparoscopic or laparotomy surgery.

The nurse is performing an assessment for a patient in menopause. What symptoms does the patient likely report having? Select all that apply. A. Osteoporosis B. Constipation C. Vision changes D. Loss of skin elasticity E. Hot flashes and irregular menses

A. Osteoporosis D. Loss of skin elasticity E. Hot flashes and irregular menses

The nurse is providing education to a group of perimenopausal women. Which herbs or supplements would the nurse include in a discussion regarding effective alternative therapies for menopausal symptoms? Select all that apply. A. Soy B. Garlic C. Ginkgo D. Vitamin A E. Black cohosh

A. Soy E. Black cohosh

A patient is determined to have a second-degree uterine prolapse. Which assessment finding by the nurse correlates with this diagnosis? A. The cervix is at the vaginal opening. B. The cervix is at lower part of the vagina. C. The vagina and rectum have no support. D. The uterus protrudes through the introitus.

A. The cervix is at the vaginal opening.

The nurse is providing education to a postmenopausal patient regarding nutritional therapy. What information provided by the nurse will have the best outcome? Select all that apply. A. Vitamin E helps to reduce hot flashes. B. Daily calorie intake should be 60 cal/kg of body weight. C. Calcium supplements must be taken two hours after meals. D. Adequate intake of calcium and vitamin D helps to maintain healthy bones. E. Women on estrogen replacement need a maximum of 1000 mg of calcium per day.

A. Vitamin E helps to reduce hot flashes. D. Adequate intake of calcium and vitamin D helps to maintain healthy bones.

The nurse is volunteering at a community center to teach women regarding breast cancer. The nurse would include which of the following when discussing risk factors (select all that apply)? A. Nulliparity B. Age 30 or over C. Early menarche D. Late menopause E. Personal history of colon cancer

ACDE. Women are at an increased risk for development of breast cancer if they are over the age of 50; have a family history of breast cancer; have a personal history of breast, colon, endometrial, or ovarian cancer; have a long menstrual history as seen with early menarche or late menopause; and have had a first full-term pregnancy after the age of 30 or are nulliparous.

Which information about continuous bladder irrigation will the nurse teach to a patient who is being admitted for a transurethral resection of the prostate (TURP)? a. Bladder irrigation decreases the risk of postoperative bleeding. b. Hydration and urine output are maintained by bladder irrigation. c. Antibiotics are infused continuously through the bladder irrigation. d. Bladder irrigation prevents obstruction of the catheter after surgery.

ANS: D The purpose of bladder irrigation is to remove clots from the bladder and to prevent obstruction of the catheter by clots. The irrigation does not decrease bleeding or improve hydration. Antibiotics are given by the IV route, not through the bladder irrigation.

The nurse is assessing four patients receiving sertraline. Which patient may benefit from the administration of this medication? A. A patient with uterine fibroids and endometrial polyps B. A patient with migraine headache before menstruation C. A patient with abdominal pain radiating to the lower back D. A patient with candidiasis with thick curdlike vaginal discharge

B. A patient with migraine headache before menstruation

The patient is being treated for a recurrent episode of chlamydia. What should the nurse include in this teaching? A. If you are treated, your sexual partner will not need to be treated. B. Abstain from sexual intercourse for seven days after finishing the treatment. C. You will probably get gonorrhea if you have another recurrence of chlamydia. D. Because you have been treated before, you do not need to take a full course of medication this time.

B. Abstain from sexual intercourse for seven days after finishing the treatment.

The nurse is educating a patient about the advantages of acyclovir therapy for herpes infection. What information should the nurse include? Select all that apply. A. Acyclovir cures herpes infection. B. Acyclovir shortens the duration of viral shedding. C. Acyclovir shortens the healing time of genital lesions. D. Acyclovir makes the patient noninfectious. E. Acyclovir reduces outbreaks of the lesions.

B. Acyclovir shortens the duration of viral shedding. C. Acyclovir shortens the healing time of genital lesions. E. Acyclovir reduces outbreaks of the lesions.

A patient is diagnosed with a vesicovaginal fistula and is to be discharged. What instructions should the nurse give to this patient? Select all that apply. A. Reduce fluid intake. B. Change perineal pads frequently. C. Take a warm sitz bath once a day. D. Cleanse the perineum every four hours. E. Perform Kegel exercises several times a day.

B. Change perineal pads frequently. D. Cleanse the perineum every four hours.

After assessing a patient with premenstrual syndrome (PMS), the nurse anticipates that the primary health care provider will prescribe ibuprofen to the patient. Which symptom of PMS does the nurse inform the patient that this will alleviate? A. Anxiety B. Cramping pain C. Fluid retention D. Negative mood

B. Cramping pain

Recurrent vaginitis may be classified as severe when a patient experiences more than how many episodes per year? A. Two B. Four C. Five D. Six

B. Four

A pregnant woman with a sexually transmitted infection (STI) is admitted in labor. Which STIs would support the need for a cesarean delivery? Select all that apply. A. Chlamydial infection B. Genital warts blocking the cervix C. Active lesions of genital herpes D. Syphilis E. Gonorrheal infection with purulent discharge from the cervix

B. Genital warts blocking the cervix C. Active lesions of genital herpes

The diagnostic reports of a patient who has infertility problems indicate the presence of tubal obstruction due to the growth of endometrial tissue. Which medication would be most beneficial for the patient? Select all that apply. A. Pergonal B. Nafarelin C. Repronex D. Humegon E. Leuprolide

B. Nafarelin E. Leuprolide

The nurse is preparing to administer medication for a patient who is experiencing premenstrual dysphoric disorder (PMDD). Which medication will the nurse anticipate administering? A. Natazia B. Sertraline C. Ibuprofen D. Methotrexate

B. Sertraline

A patient approaches the nurse expressing concerns about her impending menopause. What information should the nurse give to this patient to alleviate anxiety? Select all that apply. A. Menopause is a sudden event occurring in life. B. The age of menopause ranges from 44 to 55 years. C. Cigarette smoking may lead to early menopause. D. The age of menopause is affected by number of pregnancies. E. Menopause usually occurs as a result of illness, chemotherapy, or radiotherapy.

B. The age of menopause ranges from 44 to 55 years. C. Cigarette smoking may lead to early menopause.

The risk of infertility increases in a woman after what age? A. 20 B. 30 C. 40 D. 50

C. 40

The nurse is assessing a patient suspected of being infected with gonorrhea. What symptom elicited by a nurse assessing a male patient correlates with this suspicion? A. Scrotal edema B. Urethral discharge C. A rash on the penis D. Enlarged inguinal lymph nodes

B. Urethral discharge

Which diagnostic test does the nurse question when it is prescribed for a patient diagnosed with pelvic inflammatory disease (PID)? A. Rapid plasma reagin (RPR) B. Throat culture for streptococcus A C. Vaginal culture for Neisseria gonorrhea D. Complete blood count (CBC) with differential

B. Throat culture for streptococcus A

The nurse administers ibuprofen to a patient with premenstrual syndrome. What information does the nurse include when explaining the reasons for this prescription? A. To reduce anxiety B. To reduce cramping pain C. To reduce mood swings D. To reduce appetite

B. To reduce cramping pain

Which of the following factors may increase the severity of injuries a woman experiences following a sexual assault? Select all that apply. A. Location of the assault B. Woman's age at time of assault C. Season in which assault occurred D. Use of a weapon during the assault E. Type of clothing the woman was wearing F. The perpetrator being known to the victim

B. Woman's age at time of assault D. Use of a weapon during the assault F. The perpetrator being known to the victim

Which benign breast disorder is the most similar to breast cancer? A. Ductal ectasia B. Fibroadenoma C. Breasat abscess D. Cyclic mastalgia

C. A breast abscess is a benign breast disorder most likely to be confused with a malignany breast lesion because it shares moe characteristics of breast cancers than other benign breast disorders. A breast abscess can be red and edematous in apperance characteristic of inflammatory breast cancer. The abscess also is likely to be a palpable mass, a potential characteristic of breast cancer.

When evaluating a patient for sexually transmitted infections (STIs), the nurse is aware that which STI is the most common bacterial STI? A. Gonorrhea B. Syphilis C. Chlamydia D. Trichomoniasis

C. Chlamydia

The nurse is preparing a nutritional plan for a postmenopausal woman. What does the nurse include in the patient's dietary plan? Select all that apply. A. A low-carbohydrate diet B. A reduction in vitamin D C. Daily intake of vitamin E D. Adequate intake of calcium E. Adequate intake of sunflower seeds

C. Daily intake of vitamin E D. Adequate intake of calcium E. Adequate intake of sunflower seeds

A nurse is caring for a 36-year-old woman who has induced menopause. The nurse informs the patient that induced menopause can be attributed to which factors? Select all that apply. A. Age B. Diet C. Drugs D. Surgery E. Chemotherapy F. Radiation therapy

C. Drugs D. Surgery E. Chemotherapy F. Radiation therapy

The nurse finds that a patient reporting infertility has polycystic ovary syndrome (PCOS) that has been left untreated. About which complications does the nurse anticipate educating the patient? Select all that apply. A. Leukorrhea B. Cervical stenosis C. Diabetes mellitus D. Cardiovascular disease E. Hematogenous metastases

C. Diabetes mellitus D. Cardiovascular disease

A patient is on intravenous acyclovir for management of genital herpes-related pneumonitis. For which symptoms should the nurse look in this patient? Select all that apply. A. Disorientation B. Loss of sense of taste C. Elevated creatinine levels D. Decreased urinary output E. Peripheral edema, especially in the feet

C. Elevated creatinine levels D. Decreased urinary output E. Peripheral edema, especially in the feet

Which condition does the nurse suspect in a patient with a history of hereditary nonpolyposis colorectal cancer (HNPCC) who reports abnormal uterine bleeding? A. Leiomyomas B. Ovarian cancer C. Endometrial cancer D. Polycystic ovary syndrome

C. Endometrial cancer

Which is the biggest risk factor for endometrial cancer? A. Obesity B. Pregnancy C. Estrogen exposure D. Use of oral contraceptives

C. Estrogen exposure

The nurse is preparing a patient for a procedure that will stage ovarian cancer. For what procedure will the nurse ensure that the patient is prepared? A. OVA1 test B. Abdominal ultrasound C. Exploratory Laparotomy D. Papanicolaou (Pap) test

C. Exploratory Laparotomy

When collecting health history from a patient with chlamydial infection, what symptoms is the patient likely to report? Select all that apply. A. Bleeding from the anus B. Tenesmus C. Frequent and painful urination D. Pain during intercourse E. Menstrual abnormalities

C. Frequent and painful urination D. Pain during intercourse E. Menstrual abnormalities

The nurse is providing care for a patient who is experiencing irregular vaginal bleeding. How does the nurse document this occurrence in the patient's medical record? A. Amenorrhea B. Oligomenorrhea C. Intermenstrual bleeding D. Heavy Menstrual Bleeding (HMB)

C. Intermenstrual bleeding

A patient reports discomfort in the rectal and abdominal area and also having uterine bleeding not related to menstruation. The ultrasound reveals an enlarged uterus with nodular masses. What medication does the nurse anticipate to be administered to this patient? A. Cisplatin B. Flutamide C. Leuprolide D. Clomiphene

C. Leuprolide

A patient has been diagnosed with pelvic inflammatory disease (PID). What is a priority topic that the nurse should discuss with the patient? A. The importance of contraception B. Benefits of hormone therapy (HT) C. Manifestations of further infection D. The importance of maintaining hygiene

C. Manifestations of further infection

A patient with polycystic ovary syndrome (PCOS) is being treated to restore ovulation. What medication does the nurse administer to the patient? A. Flutamide B. Megestrol C. Metformin D. Clomiphene

C. Metformin

The nurse is caring for a patient who has an unruptured ectopic pregnancy located in the fallopian tube. What treatment option does the nurse anticipate administering? A. Leuprolide B. Nortriptyline C. Methotrexate D. Metronidazole

C. Methotrexate

A patient with uterine fibroids causing Heavy Menstrual Bleeding (HMB) informs the nurse that she wants to become pregnant. About what treatment will the nurse educate the patient that will have the best outcome? A. Cryotherapy B. Hysterectomy C. Myomectomy D. Microwave energy

C. Myomectomy

A patient reports back pain after a hysterectomy, and the nurse observes that urine output is less than 30 mL/hr. What is the priority intervention provided by the nurse? A. Administering analgesics B. Monitoring other signs and symptoms C. Notifying the primary health care provider D. Documenting the condition as normal observation

C. Notifying the primary health care provider

A patient expresses concern over her risk factors for developing ovarian cancer since her mother and sister died from the disease. What should the nurse include when teaching about early detection of ovarian cancer? A. Report any pelvic or vaginal bleeding soon B. Use estrogen with progestin for menopause C. Obtain annual bimanual pelvic examinations D. Receive a preventive bilateral oophorectomy

C. Obtain annual bimanual pelvic examinations

A patient reports to the nurse that they have feelings of "being full quickly", weight gain, menstrual changes. The nurse assesses an increase in abdominal girth from a previous visit last month. What does the nurse infer from these findings? A. Vulvar cancer B. Cervical cancer C. Ovarian cancer D. Endometrial cancer

C. Ovarian cancer

A nurse is educating a patient on abnormal uterine bleeding. The nurse should encourage the patient to report which symptoms to her healthcare provider? Select all that apply. A. Insomnia B. Weight gain C. Passing clots D. Excessive bleeding E. Spotting between regular menstrual cycles F. Unusually long duration of menstrual cycles

C. Passing clots D. Excessive bleeding F. Unusually long duration of menstrual cycles

The nurse is educating a patient with genital warts about treatment options. Which treatment options are patient-managed? Select all that apply. A. 80% to 90% trichloroacetic acid B. Podophyllin resin (10% to 25%) C. Podofilox (5%) liquid D. Podofilox (5%) E. Imiquimod (5%) cream

C. Podofilox (5%) liquid D. Podofilox (5%) E. Imiquimod (5%) cream

A patient has a third-degree uterine prolapse but denies observing anything unusual. The patient's husband recently passed away from pancreatic cancer and was cared for at home by the patient. What are the most plausible reasons the nurse identifies for this lack of self-awareness? Select all that apply. A. The cervix was only at the vaginal opening. B. The patient didn't attend to hygiene of the urogenital area. C. The patient was overwhelmed by caring for her husband. D. The cervix was only resting in the lower part of the vagina. E. The patient was embarrassed about not having noticed the prolapsed uterus.

C. The patient was overwhelmed by caring for her husband. E. The patient was embarrassed about not having noticed the prolapsed uterus.

There are several pregnant women ready to deliver their babies. Which woman should the nurse expect to require a cesarean section to deliver her baby? A. The woman who had contact with an individual with syphilis two weeks ago B. The woman who had treatment for gonococcal pharyngitis before conception C. The woman who has herpes simplex virus type 2 vesicles on her cervix at the time of delivery D. The woman who received treatment for Chlamydia trachomatis at her 20th week of gestation

C. The woman who has herpes simplex virus type 2 vesicles on her cervix at the time of delivery

A patient who is sexually active with multiple partners is at the clinic with symptoms of purulent vaginal discharge, dysuria, and dyspareunia. What should the nurse explain to the patient as the rationale for screening her for chlamydia? A. Chlamydia frequently is comorbid with human immunodeficiency virus (HIV). B. Chlamydial infections may progress to sepsis. C. Untreated chlamydial infections can lead to infertility. D. Chlamydial infections are treatable only in the early stages of infection.

C. Untreated chlamydial infections can lead to infertility.

A patient is diagnosed with human papillomavirus (HPV) infection. What teaching should the nurse provide to the patient? A. The importance of taking Gardasil after diagnosis. B. Being sure to take all antibiotic therapy C. Wart removal options D. Treatment with antiviral drugs

C. Wart removal options

The sexual assault nurse examiner (SANE) has been asked to facilitate a college campus orientation breakout session. Which item would the SANE cover during this breakout session? A. Do not leave your laptop open and unattended. B. Exercise daily to promote blood flow to the brain. C. Yell "Fire" if you are attacked, and run toward a lit area. D. Eat protein-rich snacks and vegetables during exam week.

C. Yell "Fire" if you are attacked, and run toward a lit area.

A clinic nurse is seeing a patient who is receiving treatment for condylomata. The patient asks the nurse, "Why is it important to get this treatment?" What is the best response by the nurse? A. "If you don't take this treatment, then it will turn into gonorrhea." B. "The human papillomavirus (HPV) also can cause you to develop a herpes outbreak." C. "Genital warts make you more likely to develop recurrent urinary tract infections (UTIs)." D. "Because this sexually transmitted infection increases the risk of developing cervical cancer."

D. "Because this sexually transmitted infection increases the risk of developing cervical cancer."

The primary health care provider has prescribed metronidazole for a woman diagnosed with trichomoniasis. Which statement should be included in the nurse's instructions? A. "It will turn your urine orange." B. "Alcohol does not need to be avoided." C. "This medication may produce drowsiness." D. "Both partners must be treated with the medication."

D. "Both partners must be treated with the medication."

The nurse is providing an educational program to a group of young women about the importance of Papanicolaou (Pap) testing for cervical cancer. Which information should the nurse include? A. Pap tests are 100 percent accurate in screening for cervical cell abnormalities. B. A Pap test should be conducted in women who have had a total hysterectomy. C. Regular Pap testing should begin at the age the woman becomes sexually active. D. A Pap test should be conducted every 3 years in women aged 21 to 29 years.

D. A Pap test should be conducted every 3 years in women aged 21 to 29 years.

The nurse collects a health history from a patient with a suspected malignancy. What finding is considered significant? A. Obesity since childhood B. Annual influenza vaccination C. Varicose veins from an early age D. History of human papillomavirus

D. History of human papillomavirus

A patient is diagnosed with stage III cervical cancer. Which is the most beneficial treatment that the nurse anticipates to be prepared for the patient? A. Cryosurgery B. Cervical conization C. Radical hysterectomy D. Cisplatin-based chemotherapy

D. Cisplatin-based chemotherapy

Upon assessment, the nurse suspects a patient on a medication therapy for polycystic ovary syndrome (PCOS) may be pregnant. Which medication does the nurse determine may have contributed to the pregnancy? A. Cervarix B. Flutamide C. Leuprolide D. Clomiphene

D. Clomiphene

Along with the Papanicolaou (Pap) test, which test is used to screen for cervical cancer? A. CA-125 test B. Colposcopy C. Cervical biopsy D. Human papillomavirus (HPV) test

D. Human papillomavirus (HPV) test

Which is the most common noninvasive modality of treatment for a leiomyoma? A. Hysterectomy B. Oophorectomy C. Radiation therapy D. Oral contraceptive pill (OCP)

D. Oral contraceptive pill (OCP)

A patient is being evaluated for dysmenorrhea. The healthcare provider just finished a complete health history. Which action does the nurse anticipate the healthcare provider will perform next? A. Lab work B. Ultrasound C. Pregnancy test D. Pelvic examination

D. Pelvic examination

What is the term for multiple small ovarian follicles that are produced as a result of ovulatory dysfunction? A. Ovarian cysts B. Ovarian cancer C. Cervical polyps D. Polycystic ovarian syndrome (PCOS)

D. Polycystic ovarian syndrome (PCOS)

Which term is used to describe premenstrual syndrome (PMS) associated with severe mood changes? A. Menarche B. Depression C. Perimenopause D. Premenstrual dysphoric disorder (PMDD)

D. Premenstrual dysphoric disorder (PMDD)

A patient reports cessation of her menstrual cycle to the nurse. Which condition does the nurse suspect in the patient after learning that the patient had menarche 15 years ago? A. Endometriosis B. Oligomenorrhea C. Primary amenorrhea D. Secondary amenorrhea

D. Secondary amenorrhea

A patient with total cholesterol of 210 mg/dL and cessation of menstruation one year ago reports easy fatigue, generalized weakness, joint pain, and night sweats. Which treatment strategy that may provide comfort will nurse educate the patient about? A. Hormonal therapy with estradiol B. Tricyclic antidepressants with spironolactone C. Nonsteroidal antiinflammatory agents with danazol D. Selective estrogen receptor modulators with risedronate

D. Selective estrogen receptor modulators with risedronate

A patient diagnosed with polycystic ovarian syndrome (PCOS) is experiencing hirsutism. About which medication does the nurse anticipate educating the patient? A. Flutamide B. Leuprolide C. Clomiphene D. Spironolactone

D. Spironolactone

A patient with newly diagnosed ovarian cancer states to the nurse, "I don't understand how the cancer spread so fast." What is the best response by the nurse? A. Ovarian cancer is symptom free. B. The ovaries are highly vascular organs. C. Cell division in ovarian cancer is rapid and uninhibited. D. The pelvic lymphatics and blood vessels are intermingled.

D. The pelvic lymphatics and blood vessels are intermingled.

A patient diagnosed with leiomyomas asks the nurse why she is having discomfort in the lower abdominal area. What is the best response by the nurse? A. There is twisting of the tumor pedicle. B. There is a lack of blood flow to the lower abdomen. C. Blood vessels are being compressed by the tumors. D. There is pressure on surrounding organs from the leiomyomas.

D. There is pressure on surrounding organs from the leiomyomas.

The nurse is caring for a patient with pelvic inflammatory disease. The nurse knows that positioning the patient into supine position with the head of the bed to 30 degrees is known to do what? A. To promote circulation B. To prevent dehydration C. To reduce pelvic inflammation D. To promote drainage of the pelvic cavity

D. To promote drainage of the pelvic cavity

The nurse is educating an adolescent female about the use of tampons. Which condition will the nurse inform the patient can result from the prolonged use of superabsorbent tampons? A. Dyspareunia B. Dysmenorrhea C. Abnormal uterine bleeding D. Toxic shock syndrome (TSS)

D. Toxic shock syndrome (TSS)

The nurse is providing care to a patient with frothy greenish and grey vaginal discharge. A pelvic examination reveals hemorrhagic spots on the cervix and vaginal walls. On the basis of these assessment data, which diagnosis does the nurse anticipate? A. Cervicitis B. Bacterial vaginosis C. Vulvovaginal candidiasis D. Trichomonas vaginitis infection

D. Trichomonas vaginitis infection

The nurse at a women's clinic is providing education to a woman with a history of a cystocele. Which symptom does the nurse include as a reason to contact the primary health care provider? A. Backache B. Constipation C. Urinary stress incontinence D. Urinary frequency and dysuria

D. Urinary frequency and dysuria

The nurse is educating a patient regarding the risk factors for ovarian cancer after the patient states that she is concerned because her sister died with the disease. What is the best response by the nurse? A. "Some forms of hormonal therapy may be a risk factor." B. "Oral contraceptive use is a risk factor for ovarian cancer." C. "Multiple pregnancies will place you at risk for ovarian cancer." D. "Breastfeeding is a risk factor for the development of ovarian cancer."

A. "Some forms of hormonal therapy may be a risk factor."

A female patient is admitted to the fertility clinic newly diagnosed with polycystic ovarian syndrome (PCOS). Which nursing action should be the priority? A. Provide emotional support for infertility. B. Address patient concerns about hirsutism. C. Explain why a lipid profile will be collected. D. Begin teaching about weight management.

A. Provide emotional support for infertility.

A 19-year-old visits the health clinic for a routine checkup. Which question should the nurse ask to determine whether a Pap test is needed? a. "Have you had sexual intercourse?" b. "Do you use any illegal substances?" c. "Do you have cramping with your periods?" d. "At what age did your menstrual periods start?"

A The current American Cancer Society recommendation is that a Pap test be done every 3 years, starting 3 years after the first sexual intercourse and no later than age 21. The information about menstrual periods and substance abuse will not help determine whether the patient requires a Pap test. DIF: Cognitive Level: Apply (application) REF: 1292-1293 TOP: Nursing Process: Assessment MSC:

The nurse is educating a patient about high-risk sexual behaviors. What should the nurse include when discussing what places them at a higher risk for contracting a sexually transmitted infection (STI)? Select all that apply. A. Greater sexual freedom B. Later reproductive maturity C. Limited number of susceptible hosts D. The media's increased emphasis on sexuality E. Substance abuse leads to unsafe sexual practices

A. Greater sexual freedom D. The media's increased emphasis on sexuality E. Substance abuse leads to unsafe sexual practices

department after a motor vehicle crash is scheduled for chest and abdominal x-rays. Which information is most important to report to the health care provider before the x-rays are obtained? a. Severity of abdominal pain b. Positive result of hCG test c. Blood pressure 172/88 mm Hg d. Temperature 102.1° F (38.9° C)

ANS: B Positive hCG testing indicates that the patient is pregnant and that unnecessary abdominal x-rays should be avoided. The other information is also important to report, but it will not affect whether the x-rays should be done

During examination of a 67-year-old man, the nurse notes bilateral enlargement of the breasts. The nurse's first action should be to a. palpate the breasts for the presence of any discrete lumps. b. explain that this is a temporary condition caused by hormonal changes. c. refer the patient for mammography and biopsy of the breast tissue. d. teach the patient about dietary changes to reduce the breast size.

Answer: A Rationale: If discrete, circumscribed lumps are present, the patient should be referred for further testing to determine whether breast cancer is present. Gynecomastia is usually a temporary change, but it can be caused by breast cancer. Mammography and biopsy will not be needed unless lumps are present in the breast tissue. Dietary changes will not affect the condition. Cognitive Level: Application Text Reference: p. 1348 Nursing Process: Implementation NCLEX: Physiological Integrity

The nurse will plan to teach a 34-year-old patient diagnosed with stage 0 cervical cancer about a. radiation. b. conization. c. chemotherapy. d. radical hysterectomy.

B Because the carcinoma is in situ, conization can be used for treatment. Radical hysterectomy, chemotherapy, or radiation will not be needed. DIF: Cognitive Level: Apply (application) REF: 1293 TOP: Nursing Process: Planning MSC:

A 32-year-old patient has oral contraceptives prescribed for endometriosis. The nurse will teach the patient to a. expect to experience side effects such as facial hair. b. take the medication every day for the next 9 months. c. take calcium supplements to prevent developing osteoporosis during therapy. d. use a second method of contraception to ensure that she will not become pregnant.

B When oral contraceptives are prescribed to treat endometriosis, the patient should take the medications continuously for 9 months. Facial hair is a side effect of synthetic androgens. The patient does not need to use additional contraceptive methods. The hormones in oral contraceptives will protect against osteoporosis. DIF: Cognitive Level: Apply (application) REF: 1290 TOP: Nursing Process: Implementation MSC:

Which patient in the women's health clinic will the nurse expect to teach about an endometrial biopsy? a. The 55-year-old patient who has 3 to 4 alcoholic drinks each day b. The 35-year-old patient who has used oral contraceptives for 15 years c. The 25-year-old patient who has a family history of hereditary nonpolyposis colorectal cancer d. The 45-year-old patient who has had 6 full-term pregnancies and 2 spontaneous abortions

C Patients with a personal or familial history of hereditary nonpolyposis colorectal cancer are at increased risk for endometrial cancer. Alcohol addiction does not increase this risk. Multiple pregnancies and oral contraceptive use offer protection from endometrial cancer. DIF: Cognitive Level: Apply (application) REF: 1294 OBJ: Special Questions: Multiple Patients TOP: Nursing Process: Assessment MSC:

Which action should the nurse take when a 35-year-old patient has a result of minor cellular changes on her Pap test? a. Teach the patient about colposcopy. b. Teach the patient about punch biopsy. c. Schedule another Pap test in 4 months. d. Administer the human papillomavirus (HPV) vaccine.

C Patients with minor changes on the Pap test can be followed with Pap tests every 4 to 6 months because these changes may revert to normal. Punch biopsy or colposcopy may be used if the Pap test shows more prominent changes. The HPV vaccine may reduce the risk for cervical cancer, but it is recommended only for ages 9 through 26. DIF: Cognitive Level: Apply (application) REF: 1293 TOP: Nursing Process: Implementation MSC:

The nurse in the infertility clinic is explaining in vitro fertilization (IVF) to a couple. The woman tells the nurse that they cannot afford IVF on her husband's salary. The man replies that if his wife worked outside the home, they would have enough money. Which nursing diagnosis is appropriate? a. Decisional conflict related to inadequate financial resources b. Ineffective sexuality patterns related to psychological stress c. Defensive coping related to anxiety about lack of conception d. Ineffective denial related to frustration about continued infertility

C The statements made by the couple are consistent with the diagnosis of defensive coping. No data indicate that ineffective sexuality and ineffective denial are problems. Although the couple is quarreling about finances, the data do not provide information indicating that the finances are inadequate. DIF: Cognitive Level: Apply (application) REF: 1278 TOP: Nursing Process: Diagnosis MSC:

A patient has a speculum examination that reveals bright cherry-red pedunculated lesions on the cervix. Which condition is associated with these assessment findings? A. Leiomyomas B. Cervical cancer C. Cervical polyps D. Endometrial cancer

C. Cervical polyps

A couple who failed to conceive under infertility management has been asked to go for in vitro fertilization (IVF). When explaining the procedure, what information should the nurse include? Select all that apply. A. The procedure requires two to three hours. B. The procedure cannot be done if the partner has low sperm count. C. This procedure is done for patients with tubal obstruction and diminished sperm count. D. This procedure is financially costly, but a single attempt provides successful implantation. E. With this procedure, the embryo develops outside the body and is later transplanted into the patient's uterus.

C. This procedure is done for patients with tubal obstruction and diminished sperm count. E. With this procedure, the embryo develops outside the body and is later transplanted into the patient's uterus.

When providing teaching to a patient with gonorrhea, which statement is indicative that further teaching is needed? A. "I'll take the medication until the discharge is gone." B. "I will wear cotton panties rather than a thong." C. "I will skip sex for the next week, then use a condom." D. "My partner has to come to the clinic to be treated too."

A. "I'll take the medication until the discharge is gone."

A patient is diagnosed with a cervical polyp. What information should the nurse give this patient? Select all that apply. A. A malignant change may occur. B. Polyps may be excised or cauterized on an outpatient basis. C. A single polyp is usually benign; multiple polyps are malignant. D. Polyps are due to infection and a course of antibiotics is needed. E. Polyps are due to aggressive intercourse, and intercourse should be avoided for two weeks.

A. A malignant change may occur. B. Polyps may be excised or cauterized on an outpatient basis.

The nurse is assessing the sexual-reproductive functional health pattern of a 32-year-old woman. Which question is most useful in determining the patient's sexual orientation and risk factors? a. "Do you have sex with men, women, or both?" b. "Which gender do you prefer to have sex with?" c. "What types of sexual activities do you prefer?" d. "Are you heterosexual, homosexual, or bisexual?"

ANS: A This question is the most simply stated and will increase the likelihood of obtaining the relevant information about sexual orientation and possible risk factors associated with sexual activity. A patient who prefers sex with women may also have intercourse at times with men. The types of sexual activities engaged in may not indicate sexual orientation. Many patients who have sex with both men and women do not identify themselves as homosexual or bisexual.

The nurse is providing teaching by telephone to a patient who is scheduled for a pelvic examination and Pap test next week. The nurse instructs the patient that she should a. shower, but not take a tub bath, before the examination. b. not have sexual intercourse the day before the Pap test. c. avoid douching for at least 24 hours before the examination. d. schedule to have the Pap test just after her menstrual period.

ANS: C Because the results of a Pap test may be affected by douching, the patient should not douche before the examination. The exam may be scheduled without regard to the menstrual period. The patient may shower or bathe before the examination. Sexual intercourse does not affect the results of the examination or Pap test.

Which assessment information is most important for the nurse to report to the health care provider when a patient asks for a prescription for testosterone replacement therapy (TRT)? a. The patient has noticed a decrease in energy level for a few years. b. The patient's symptoms have increased steadily over the last few years. c. The patient has been using sildenafil (Viagra) several times every week. d. The patient has had a gradual decrease in the force of his urinary stream.

ANS: D The decrease in urinary stream may indicate benign prostatic hyperplasia (BPH) or prostate cancer, which are contraindications to the use of testosterone replacement therapy (TRT). The other patient data indicate that TRT may be a helpful therapy for the patient

When assessing a patient for breast cancer risk, the nurse considers that the patient has a significant family history of breast cancer if she has a a. cousin who was diagnosed with breast cancer at age 38. b. mother who was diagnosed with breast cancer at age 42. c. sister who died from ovarian cancer at age 56. d. grandmother who died from breast cancer at age 72.

Answer: B Rationale: A significant family history of breast cancer means that the patient has a first-degree relative who developed breast cancer, especially if the relative was premenopausal. Cognitive Level: Application Text Reference: p. 1348 Nursing Process: Assessment NCLEX: Health Promotion and Maintenance

A patient with a breast biopsy positive for cancer is to undergo lymphatic mapping and sentinel lymph node dissection (SLND). The nurse explains that this procedure a. can identify specific lymph nodes that have malignant cells, so only involved nodes need to be excised. b. reduces the need for extensive lymph node dissection for pathologic examination. c. eliminates the need for excision of more than one lymph node for staging of breast cancer. d. will confirm the absence of tumor spread if the sentinel lymph node is negative for malignant changes.

Answer: B Rationale: The SLND may eliminate further lymph node dissection if the initial nodes are negative for malignancy. The procedure identifies which lymph nodes drain first from the tumor site, but not which ones are malignant. Several lymph nodes may be dissected for pathologic examination. Tumor may have distant metastases even when no malignancies are found in the lymph nodes. Cognitive Level: Comprehension Text Reference: p. 1351 Nursing Process: Implementation NCLEX: Physiological Integrity

A woman with a positive biopsy for breast cancer is considering whether to have a modified radical mastectomy or breast conservation surgery (lumpectomy) with radiation therapy. Which information should the nurse provide? a. The postoperative survival rate for each is about the same, but there is a decreased rate of cancer recurrence after mastectomy. b. The lumpectomy and radiation will preserve the breast, but this method can cause changes in breast sensitivity. c. The hair loss associated with post-lumpectomy chemotherapy is not acceptable to some patients. d. The treatment period for the mastectomy is shorter, and breast reconstruction can provide a normal-appearing breast.

Answer: B Rationale: The impact on breast function and appearance is less with lumpectomy and radiation, but there is some effect on breast sensitivity. The rate of cancer recurrence is the same for the two procedures. Chemotherapy may be used after either lumpectomy or mastectomy, but it is not always needed. The treatment period is shorter after mastectomy, but breast reconstruction does not provide a normal-appearing breast. Cognitive Level: Application Text Reference: pp. 1352-1353 Nursing Process: Implementation NCLEX: Physiological Integrity

A 31-year-old patient who has been diagnosed with human papillomavirus (HPV) infection gives a health history that includes smoking tobacco, taking oral contraceptives, and having been treated twice for vaginal candidiasis. Which topic will the nurse include in patient teaching? a. Use of water-soluble lubricants b. Risk factors for cervical cancer c. Antifungal cream administration d. Possible difficulties with conception

B Because HPV infection and smoking are both associated with increased cervical cancer risk, the nurse should emphasize the importance of avoiding smoking. An HPV infection does not decrease vaginal lubrication, decrease ability to conceive, or require the use of antifungal creams. DIF: Cognitive Level: Apply (application) REF: 1292 TOP: Nursing Process: Implementation MSC:

A 25-year-old woman has an induced abortion with suction curettage at an ambulatory surgical center. Which instructions will the nurse include when discharging the patient? a. "Heavy vaginal bleeding is expected for about 2 weeks." b. "You should abstain from sexual intercourse for 2 weeks." c. "Contraceptives should be avoided until your reexamination." d. "Irregular menstrual periods are expected for the next few months."

B Because infection is a possible complication of this procedure, the patient is advised to avoid intercourse until the reexamination in 2 weeks. Patients may be started on contraceptives on the day of the procedure. The patient should call the doctor if heavy vaginal bleeding occurs. No change in the regularity of the menstrual periods is expected. DIF: Cognitive Level: Apply (application) REF: 1279 TOP: Nursing Process: Implementation MSC:

A 32-year-old who was admitted to the emergency department with severe abdominal pain is diagnosed with an ectopic pregnancy. The patient begins to cry and asks the nurse to leave her alone to grieve. Which action should the nurse take next? a. Stay with the patient and encourage her to discuss her feelings. b. Explain the reason for taking vital signs every 15 to 30 minutes. c. Close the door to the patient's room and minimize disturbances. d. Provide teaching about options for termination of the pregnancy.

B Because the patient is at risk for rupture of the fallopian tube and hemorrhage, frequent monitoring of vital signs is needed. The patient has asked to be left alone, so staying with her and encouraging her to discuss her feelings are inappropriate actions. Minimizing contact with her and closing the door of the room is unsafe because of the risk for hemorrhage. Because the patient has requested time to grieve, it would be inappropriate to provide teaching about options for pregnancy termination. DIF: Cognitive Level: Apply (application) REF: 1298 TOP: Nursing Process: Implementation MSC:

A 29-year-old patient who is trying to become pregnant asks the nurse how to determine when she is most likely to conceive. The nurse explains that a. ovulation is unpredictable unless there are regular menstrual periods. b. ovulation prediction kits provide accurate information about ovulation. c. she will need to bring a specimen of cervical mucus to the clinic for testing. d. she should take her body temperature daily and have intercourse when it drops.

B Ovulation prediction kits indicate when luteinizing hormone (LH) levels first rise. Ovulation occurs about 28 to 36 hours after the first rise of LH. This information can be used to determine the best time for intercourse. Body temperature rises at ovulation. Postcoital cervical smears are used in infertility testing, but they do not predict the best time for conceiving and are not obtained by the patient. Determination of the time of ovulation can be predicted by basal body temperature charts or ovulation prediction kits and is not dependent on regular menstrual periods. DIF: Cognitive Level: Apply (application) REF: 1277 TOP: Nursing Process: Implementation MSC:

A healthy 28-year-old who has been vaccinated against human papillomavirus (HPV) has a normal Pap test. Which information will the nurse include in patient teaching when calling the patient with the results of the Pap test? a. You can wait until age 30 before having another Pap test. b. Pap testing is recommended every 3 years for women your age. c. No further Pap testing is needed until you decide to become pregnant. d. Yearly Pap testing is suggested for women with multiple sexual partners.

B The current national guidelines suggest Pap testing every 3 years for patients between ages 21 to 65. Although HPV immunization does protect against cervical cancer, the recommendations are unchanged for individuals who have received the HPV vaccination. DIF: Cognitive Level: Understand (comprehension) REF: 1293 TOP: Nursing Process: Planning MSC:

A 32-year-old woman is scheduled for an induced abortion using instillation of hypertonic saline solution. Which information will the nurse plan to discuss with the patient before the procedure? a. The patient will require a general anesthetic. b. The expulsion of the fetus may take 1 to 2 days. c. There is a possibility that the patient may deliver a live fetus. d. The procedure may be unsuccessful in terminating the pregnancy.

B Uterine contractions take 12 to 36 hours to begin after the hypertonic saline is instilled. Because the saline is feticidal, the nurse does not need to discuss any possibility of a live delivery or that the pregnancy termination will not be successful. General anesthesia is not needed for this procedure. DIF: Cognitive Level: Apply (application) REF: 1278 TOP: Nursing Process: Implementation MSC:

Which nursing actions can the nurse working in a women's health clinic delegate to unlicensed assistive personnel (UAP) (select all that apply)? a. Call a patient with the results of an endometrial biopsy. b. Assist the health care provider with performing a Pap test. c. Draw blood for CA-125 levels for a patient with ovarian cancer. d. Screen a patient for use of medications that may cause amenorrhea. e. Teach the parent of a 10-year-old about the human papilloma virus (HPV) vaccine (Gardasil).

B, C Assisting with a Pap test and drawing blood (if trained) are skills that require minimal critical thinking and judgment and can be safely delegated to UAP. Patient teaching, calling a patient who may have questions about results of diagnostic testing, and risk-factor screening all require more education and critical thinking and should be done by the registered nurse (RN). DIF: Cognitive Level: Apply (application) REF: 15-16 OBJ: Special Questions: Delegation TOP: Nursing Process: Planning MSC:

The nurse is educating a teenage patient about risk factors for cancer of the reproductive system. Which patient statements demonstrate understanding of the education? Select all that apply. A. "I will avoid alcohol." B. "I will avoid smoking." C. "I will increase my intake of dietary fiber." D. "I will use condoms for sexual intercourse." E. "I will limit sexual activity in my adolescent years." F. "I will urinate as soon as sexual intercourse is over."

B. "I will avoid smoking." D. "I will use condoms for sexual intercourse." E. "I will limit sexual activity in my adolescent years."

A nurse is assessing the risk behaviors for the spread of sexually transmitted infections (STIs) in a group of people. Which patients are at high risk of contracting an STI? Select all that apply. A. A woman who has not had a sexual relationship B. A man having sexual relationships with multiple partners C. A woman having polygamous relationships and who is on oral contraceptive pills D. A man having a monogamous relationship with his partner and using a condom E. A woman having a monogamous relationship, whose partner has a history of a recent STI F. A man having a monogamous relationship who uses condoms but often consumes drugs with shared needles

B. A man having sexual relationships with multiple partners C. A woman having polygamous relationships and who is on oral contraceptive pills E. A woman having a monogamous relationship, whose partner has a history of a recent STI F. A man having a monogamous relationship who uses condoms but often consumes drugs with shared needles

Which of the following tasks should the registered nurse delegate to nursing assistive personnel (NAP) during the care of a patient who has had recent transverse rectus abdominis musculocutaneous (TRAM) flap surgery? A. Document the condition of the patient's incisions. B. Mobilize the patient in a slightly hunched position. C. Change the patient's abdominal and chest dressings. D. Change the parameters of the patient-controlled analgesic (PCA) pump.

B. Mobilization of a postsurgical patient may be delegated, and the patient who has had a TRAM flap should not stand or walk fully erect, in order to minimize strain on the incisions. Changing dressings, assessing wounds, and reprogramming a PCA pump are not tasks amenable to delegation.

The health care provider advises expedited partner therapy (EPT) to a patient with a sexually transmitted infection (STI). What should the nurse explain to this patient about EPT? A. The patient should cut off all ties with the partner because the patient will become reinfected. B. Prescription or medications can be given to the patient's partner; no examination is required. C. The partner of the concerned patient will be examined and will be advised if treatment is needed. D. The partner of the patient will be given a vaccine to prevent transmission of the sexually transmitted infection.

B. Prescription or medications can be given to the patient's partner; no examination is required.

The nurse is caring for a patient who has had a hysterectomy. Which nursing action is useful in preventing strain on the suture line? A. Ambulating the patient B. Using a indwelling catheter C. Restricting foods and fluids D. Avoiding the high-Fowler's position

B. Using a indwelling catheter

The nurse is caring for a 20-year-old patient with pelvic inflammatory disease (PID) requiring hospitalization. Which nursing intervention will be included in the plan of care? a. Monitor liver function tests. b. Use cold packs PRN for pelvic pain. c. Elevate the head of the bed to at least 30 degrees. d. Teach the patient how to perform Kegel exercises.

C The head of the bed should be elevated to at least 30 degrees to promote drainage of the pelvic cavity and prevent abscess formation higher in the abdomen. Although a possible complication of PID is acute perihepatitis, liver function tests will remain normal. There is no indication for increased fluid intake. Application of heat is used to reduce pain. Kegel exercises are not helpful in PID. DIF: Cognitive Level: Apply (application) REF: 1288 TOP: Nursing Process: Planning MSC:

A nursing diagnosis that is likely to be appropriate for a 67-year-old woman who has just been diagnosed with stage III ovarian cancer is a. sexual dysfunction related to loss of vaginal sensation. b. risk for infection related to impaired immune function. c. anxiety related to cancer diagnosis and need for treatment decisions. d. situational low self-esteem related to guilt about delaying medical care.

C The patient with stage III ovarian cancer is likely to be anxious about the poor prognosis and about the need to make decisions about the multiple treatments that may be used. Decreased vaginal sensation does not occur with ovarian cancer. The patient may develop immune dysfunction when she receives chemotherapy, but she is not currently at risk. It is unlikely that the patient has delayed seeking medical care because the symptoms of ovarian cancer are vague and occur late in the course of the cancer. DIF: Cognitive Level: Apply (application) REF: 1298 TOP: Nursing Process: Analysis MSC:

A female patient tells the nurse that she has been having nightmares and acute anxiety around men since being sexually assaulted 3 months ago. The most appropriate nursing diagnosis for the patient is a. anxiety related to effects of being raped. b. sleep deprivation related to frightening dreams. c. rape-trauma syndrome related to rape experience. d. ineffective coping related to inability to resolve incident.

C The patient's symptoms are most consistent with the nursing diagnosis of rape-trauma syndrome. The nursing diagnoses of sleep deprivation, ineffective coping, and anxiety address some aspects of the patient's symptoms but do not address the problem as completely as the rape-trauma syndrome diagnosis. DIF: Cognitive Level: Apply (application) REF: 1302 TOP: Nursing Process: Diagnosis MSC:

A patient has pelvic organ prolapse and is instructed to perform Kegel exercises. What does the nurse instruct the patient about Kegel exercises for an effective outcome? A. "You should perform exercise once a day." B. "You should avoid voiding before performing the exercises." C. "You should pull in as if you are trying to stop the urine flow." D. "You should breathe continuously while performing the exercises."

C. "You should pull in as if you are trying to stop the urine flow."

A woman is scheduled for an induced abortion on her 40th day of gestation. What instructions should the nurse give to the patient about the procedure? Select all that apply. A. Contraception can be started a month after the procedure. B. The abortion drugs must be given within the first 59 days of pregnancy. C. Fever and foul-smelling drainage postprocedure may indicate complications. D. Abnormal vaginal bleeding and severe abdominal cramping need to be reported to the health care provider. E. Intercourse and vaginal insertions should be avoided until postprocedure reexamination, which needs to be in two weeks.

C. Fever and foul-smelling drainage postprocedure may indicate complications. D. Abnormal vaginal bleeding and severe abdominal cramping need to be reported to the health care provider. E. Intercourse and vaginal insertions should be avoided until postprocedure reexamination, which needs to be in two weeks.

Menopause is the result of a decrease in which hormone? A. Estrogen B. Progesterone C. Prostaglandin D. Human chorionic gonadotropin (HCG)

C. Prostaglandin

The nurse is caring for a patient with a diagnosis of cervical cancer spreading to the pelvic wall and lower vagina. Which advanced stage of cervical cancer does the nurse suspect in this patient? A. Stage I B. Stage II C. Stage III D. Stage IV

C. Stage III

A 49-year-old woman tells the nurse that she is postmenopausal but has occasional spotting. Which initial response by the nurse is most appropriate? a. "A frequent cause of spotting is endometrial cancer." b. "How long has it been since your last menstrual period?" c. "Breakthrough bleeding is not unusual in women your age." d. "Are you using prescription hormone replacement therapy?"

D In postmenopausal women, a common cause of spotting is hormone therapy (HT). Because breakthrough bleeding may be a sign of problems such as cancer or infection, the nurse would not imply that this is normal. The length of time since the last menstrual period is not relevant to the patient's symptoms. Although endometrial cancer may cause spotting, this information is not appropriate as an initial response. DIF: Cognitive Level: Apply (application) REF: 1281 | 1284 TOP: Nursing Process: Implementation MSC:

A 28-year-old patient was recently diagnosed with polycystic ovary syndrome. It is most important for the nurse to teach the patient a. reasons for a total hysterectomy. b. how to decrease facial hair growth. c. ways to reduce the occurrence of acne. d. methods to maintain appropriate weight.

D Obesity exacerbates the problems associated with polycystic ovary syndrome, such as insulin resistance and type 2 diabetes. The nurse should also address the problems of acne and hirsutism, but these symptoms are lower priority because they do not have long-term health consequences. Although some patients do require total hysterectomy, this is usually performed only after other therapies have been unsuccessful. DIF: Cognitive Level: Apply (application) REF: 1292 OBJ: Special Questions: Prioritization TOP: Nursing Process: Implementation MSC:

The nurse has instructed a male patient who has gonorrhea about self-care. Which statement made by the patient indicates the need for further teaching? A. "I should ask my partner to get tested." B. "I need to take the antibiotics until they're finished." C. "I can keep taking showers while I'm getting treated." D. "I can have sex tonight as long as I make sure to wear a condom."

D. "I can have sex tonight as long as I make sure to wear a condom."

What instruction does the nurse give to the patient with cystocele who undergoes an anterior colporrhaphy to ensure effective treatment after surgery? Select all that apply. A. "You should eat a low-residue diet." B. "You should avoid voiding frequently." C. "You should take a stool softener every night." D. "You should avoid frequent change of positions." E. "You should avoid intercourse until the follow up visit."

A. "You should eat a low-residue diet." C. "You should take a stool softener every night." E. "You should avoid intercourse until the follow up visit."

A patient with urinary obstruction from benign prostatic hyperplasia (BPH) tells the nurse, "My symptoms are much worse this week." Which response by the nurse is most appropriate? a. "Have you been taking any over-the-counter (OTC) medications recently?" b. "I will talk to the doctor about ordering a prostate specific antigen (PSA) test." c. "Have you talked to the doctor about surgery such as transurethral resection of the prostate (TURP)?" d. "The prostate gland changes in size from day to day, and this may be making your symptoms worse."

ANS: A Because the patient's increase in symptoms has occurred abruptly, the nurse should ask about OTC medications that might cause contraction of the smooth muscle in the prostate and worsen obstruction. The prostate gland does not vary in size from day to day. A TURP may be needed, but more assessment about possible reasons for the sudden symptom change is a more appropriate first response by the nurse. PSA testing is done to differentiate BPH from prostatic cancer

A patient is suspected to have vulvar cancer. For which diagnostic study does the nurse prepare the patient? A. Colposcopy B. Laparotomy C. CA-125 test D. Papanicolaou (Pap) test

A. Colposcopy

A woman with chlamydial infection complains of pelvic pain, nausea, vomiting, fever, and abnormal vaginal bleeding. What might be the possible cause of these symptoms? A. Pelvic inflammatory disease B. Ectopic pregnancy C. Infertility D. Reactive arthritis

A. Pelvic inflammatory disease

A patient comes to the clinic with a chancre on the penis. Which diagnostic test does the nurse anticipate preparing the patient for? A. Rapid plasma region test B. Human papillomavirus DNA tests C. Treponema pallidum particle agglutination test D. Fluorescent treponemal antibody absorption test

A. Rapid plasma region test

While caring for a patient with irritability, abdominal bloating, breast discomfort, and edema, the nurse administers spironolactone. What rationale does the nurse give to the patient for administration of the medication? A. To reduce fluid retention B. To reduce cramping pain C. To improve negative mood D. To alleviate physiologic symptoms

A. To reduce fluid retention

The nurse is giving discharge instructions to a woman that had an induced abortion. What should the nurse include in the education to avoid complications related to the procedure? Select all that apply A. Report fever B. Report feelings of sadness C. Report abnormal vaginal bleeding D. Report severe abdominal cramping E. Report any foul smelling discharge

A. Report fever C. Report abnormal vaginal bleeding D. Report severe abdominal cramping E. Report any foul smelling discharge

The nurse is caring for a patient who has been sexually assaulted. What nursing actions are appropriate for this patient? Select all that apply. A. Refer to crisis center. B. Provide privacy and emotional support. C. Train her about self-defense techniques. D. Provide the primary care and admit the patient in hospital. E. Discuss the possibility of pregnancy and offer effective emergency contraceptive pills.

A. Refer to crisis center. B. Provide privacy and emotional support. E. Discuss the possibility of pregnancy and offer effective emergency contraceptive pills.

What is the nurse's priority assessment in a 41 year old female paitent who reports a lump in the upper, outer margin of the right breast? A. Related pain B. Family history C. Onset of mass D. Characteristics

A. Breast cancer is usually associated with nonpainful and nontender lesions of the breast, so this information is very important in assessing breast masses

A patient is diagnosed with polycystic ovary syndrome and asks the nurse how she could have developed this disorder. What is the best response by the nurse? A. "The disorder may result from hormonal abnormalities." B. "Taking immunosuppressive medications can cause it." C. "Obesity is an associated factor in the development of the disorder." D. "Contracting an infection caused by the human papillomavirus (HPV) is the cause."

A. "The disorder may result from hormonal abnormalities."

The nurse is caring for a patient who has experienced a tubal rupture as the result of an ectopic pregnancy. Which nursing interventions will help reduce the patient's risk for hypovolemia? Select all that apply. A. Administering intravenous fluid B. Preparing for blood transfusions C. Applying heat to the lower abdomen D. Recording the characteristics of the vaginal discharge E. Asking the patient to sleep in the semi-Fowler's position

A. Administering intravenous fluid B. Preparing for blood transfusions

The nurse suspects that the tumor of a patient with cervical cancer has enlarged, resulting in heavy bleeding. Which other late stage symptoms does the nurse expect in this patient? Select all that apply. A. Anemia B. Cachexia C. Weight loss D. Heavy bleeding E. Abdominal girth

A. Anemia B. Cachexia C. Weight loss

A patient has been scheduled to receive external radiotherapy of the bladder. What instruction should the nurse give to this patient? A. Urinate before the treatment is initiated. B. Tell visitors to stay six feet away from the bed and not to visit as often. C. Drink adequate water so that the bladder is full before initiating therapy. D. Consult the primary health care provider if symptoms of enteritis or cystitis occur.

A. Urinate before the treatment is initiated.

A patient had successful treatment for an existing sexually transmitted infection (STI). What instructions should the nurse provide to this patient to prevent reinfection or complications? Select all that apply. A. Use condoms. B. Avoid using tampons. C. Use an intrauterine device. D. Avoid having multiple sexual partners. E. Make sure to get periodic Pap smears.

A. Use condoms. D. Avoid having multiple sexual partners. E. Make sure to get periodic Pap smears.

After scheduling a patient with a possible ovarian cyst for ultrasound, the nurse will teach the patient that she should a. expect to receive IV contrast during the procedure. b. drink several glasses of fluids before the procedure. c. experience mild abdominal cramps after the procedure. d. discontinue taking aspirin for 7 days before the procedure.

ANS: B A full bladder is needed for many ultrasound procedures, so the nurse will have the patient drink fluids before arriving for the ultrasound. The other instructions are not accurate for this procedure

The nurse will plan to provide teaching for a 67-year-old patient who has been diagnosed with orchitis about a. pain management. b. emergency surgical repair. c. application of heat to the scrotum. d. aspiration of fluid from the scrotal sac.

ANS: A Orchitis is very painful, and effective pain management will be needed. Heat, aspiration, and surgery are not used to treat orchitis

During which stage of cervical cancer is the cancer confined to the cervix? A. 0 B. I C. II D. III

B. I

The nurse notes that a patient who has a large cystocele, admitted 10 hours ago, has not yet voided. Which action should the nurse take first? a. Insert a straight catheter per the PRN order. b. Encourage the patient to increase oral fluids. c. Notify the health care provider of the inability to void. d. Use an ultrasound scanner to check for urinary retention.

D Because urinary retention is common with a large cystocele, the nurse's first action should be to use an ultrasound bladder scanner to check for the presence of urine in the bladder. The other actions may be appropriate, depending on the findings with the bladder scanner. DIF: Cognitive Level: Apply (application) REF: 1300-1301 OBJ: Special Questions: Prioritization TOP: Nursing Process: Implementation MSC:

The nurse is performing an assessment on an older adult female patient. Which symptom does the nurse teach her to report to the primary health care provider? A. Dyspareunia B. Vaginal bleeding C. Skin becomes thinner D. Breasts become smaller

B. Vaginal bleeding

A patient informs the nurse that she is having heavy menstrual cycles and also would like to avoid pregnancy at this time. What medication does the nurse anticipate educating the patient about? A. Sertraline B. Paroxetine C. Alendronate D. Estradiol valerate

D. Estradiol valerate

A 56-year-old woman is concerned about having a moderate amount of vaginal bleeding after 4 years of menopause. The nurse will anticipate teaching the patient about a. endometrial biopsy. b. endometrial ablation. c. uterine balloon therapy. d. dilation and curettage (D&C).

A A postmenopausal woman with vaginal bleeding should be evaluated for endometrial cancer, and endometrial biopsy is the primary test for endometrial cancer. D&C will be needed only if the biopsy does not provide sufficient information to make a diagnosis. Endometrial ablation and balloon therapy are used to treat menorrhagia, which is unlikely in this patient. DIF: Cognitive Level: Apply (application) REF: 1294 TOP: Nursing Process: Planning MSC:

Which statement by the patient indicates that the nurse's teaching about treating vaginal candidiasis has been effective? a. "I should clean carefully after each urination and bowel movement." b. "I can douche with warm water if the itching continues to bother me." c. "I will insert the antifungal cream right before I get up in the morning." d. "I will tell my husband that we cannot have intercourse for the next month."

A Cleaning of the perineal area will decrease itching caused by contact of the irritated tissues with urine and reduce the chance of further infection of irritated tissues by bacteria in the stool. Sexual intercourse should be avoided for 1 week. Douching will disrupt normal protective mechanisms in the vagina. The cream should be used at night so that it will remain in the vagina for longer periods of time. DIF: Cognitive Level: Apply (application) REF: 1287 TOP: Nursing Process: Evaluation MSC:

A 58-year-old patient who has undergone a radical vulvectomy for vulvar carcinoma returns to the medical-surgical unit after the surgery. The priority nursing diagnosis for the patient at this time is a. risk for infection related to contact of the wound with urine and stool. b. self-care deficit: bathing/hygiene related to pain and difficulty moving. c. imbalanced nutrition: less than body requirements related to low-residue diet. d. risk for ineffective sexual pattern related to disfiguration caused by the surgery.

A Complex and meticulous wound care is needed to prevent infection and delayed wound healing. The other nursing diagnoses may also be appropriate for the patient but are not the highest priority immediately after surgery. DIF: Cognitive Level: Apply (application) REF: 1299 OBJ: Special Questions: Prioritization TOP: Nursing Process: Analysis MSC:

A 28-year-old patient with endometriosis asks why she is being treated with medroxyprogesterone (Depo-Provera), a medication that she thought was an oral contraceptive. The nurse explains that this therapy a. suppresses the menstrual cycle by mimicking pregnancy. b. will relieve symptoms such as vaginal atrophy and hot flashes. c. prevents a pregnancy that could worsen the menstrual bleeding. d. will lead to permanent suppression of abnormal endometrial tissues.

A Depo-Provera induces a pseudopregnancy, which suppresses ovulation and causes shrinkage of endometrial tissue. Menstrual bleeding does not occur during pregnancy. Vaginal atrophy and hot flashes are caused by synthetic androgens such as danazol or gonadotropin-releasing hormone agonists (GNRH) such as leuprolide. Although hormonal therapies will control endometriosis while the therapy is used, endometriosis will recur once the menstrual cycle is reestablished. DIF: Cognitive Level: Apply (application) REF: 1290 TOP: Nursing Process: Implementation MSC:

The nurse will plan to teach the female patient with genital warts about the a. importance of regular Pap tests. b. increased risk for endometrial cancer. c. appropriate use of oral contraceptives. d. symptoms of pelvic inflammatory disease (PID).

A Genital warts are caused by the human papillomavirus (HPV) and increase the risk for cervical cancer. There is no indication that the patient needs teaching about PID, oral contraceptives, or endometrial cancer. DIF: Cognitive Level: Apply (application) REF: 1286 | 1292 TOP: Nursing Process: Planning MSC:

A 50-year-old patient is diagnosed with uterine bleeding caused by a leiomyoma. Which information will the nurse include in the patient teaching plan? a. The symptoms may decrease after the patient undergoes menopause. b. The tumor size is likely to increase throughout the patient's lifetime. c. Aspirin or acetaminophen may be used to control mild to moderate pain. d. The patient will need frequent monitoring to detect any malignant changes.

A Leiomyomas appear to depend on ovarian hormones and will atrophy after menopause, leading to a decrease in symptoms. Aspirin use is discouraged because the antiplatelet effects may lead to heavier uterine bleeding. The size of the tumor will shrink after menopause. Leiomyomas are benign tumors that do not undergo malignant changes. DIF: Cognitive Level: Apply (application) REF: 1290 TOP: Nursing Process: Planning MSC:

A 22-year-old tells the nurse that she has not had a menstrual period for the last 2 months. Which action is most important for the nurse to take? a. Obtain a urine specimen for a pregnancy test. b. Ask about any recent stressful lifestyle changes. c. Measure the patient's current height and weight. d. Question the patient about prescribed medications.

A Pregnancy should always be considered a possible cause of amenorrhea in women of childbearing age. The other actions are also appropriate, but it is important to check for pregnancy in this patient because pregnancy will require rapid implementation of actions to promote normal fetal development such as changes in lifestyle, folic acid intake, etc. DIF: Cognitive Level: Apply (application) REF: 1281 OBJ: Special Questions: Prioritization TOP: Nursing Process: Assessment MSC:

A 48-year-old woman in the emergency department reports that she has been sexually assaulted. Which action by the nurse will be most important in maintaining the medicolegal chain of evidence? a. Labeling all specimens and other materials obtained from the patient. b. Assisting the patient in filling out the application for financial compensation. c. Discussing the availability of the "morning-after pill" for pregnancy prevention. d. Educating the patient about baseline sexually transmitted infection (STI) testing.

A The careful labeling of specimens and materials will assist in maintaining the chain of evidence. Assisting with paperwork, and discussing STIs and pregnancy prevention are interventions that might be appropriate after sexual assault, but they do not help maintain the legal chain of evidence. DIF: Cognitive Level: Apply (application) REF: 1302 TOP: Nursing Process: Implementation MSC:

A 32-year-old woman brought to the emergency department reports being sexually assaulted. The patient is confused about where she is and she has a large laceration above the right eye. Which action should the nurse take first? a. Assess the patient's neurologic status. b. Assist the patient to remove her clothing. c. Contact the sexual assault nurse examiner (SANE). d. Ask the patient to describe what occurred during the assault.

A The first priority is to treat urgent medical problems associated with the sexual assault. The patient's head injury may be associated with a head trauma such as a skull fracture or subdural hematoma. Therefore her neurologic status should be assessed first. The other nursing actions are also appropriate, but they are not as high in priority as assessment and treatment for acute physiologic injury. DIF: Cognitive Level: Apply (application) REF: 1302 OBJ: Special Questions: Prioritization TOP: Nursing Process: Implementation MSC:

A 28-year-old patient reports anxiety, headaches with dizziness, and abdominal bloating occurring before her menstrual periods. Which action is best for the nurse to take at this time? a. Ask the patient to keep track of her symptoms in a diary for 3 months. b. Suggest that the patient try aerobic exercise to decrease her symptoms. c. Teach the patient about appropriate lifestyle changes to reduce premenstrual syndrome (PMS) symptoms. d. Advise the patient to use nonsteroidal antiinflammatory drugs (NSAIDs) such as ibuprofen (Advil) to control symptoms.

A The patient's symptoms indicate possible PMS, but they also may be associated with other diagnoses. Having the patient keep a symptom diary for 2 or 3 months will help in confirming a diagnosis of PMS. The nurse should not implement interventions for PMS until a diagnosis is made. DIF: Cognitive Level: Apply (application) REF: 1279 TOP: Nursing Process: Implementation MSC:

Which topic will the nurse include in the preoperative teaching for a patient admitted for an abdominal hysterectomy? a. Purpose of ambulation and leg exercises b. Adverse effects of systemic chemotherapy c. Decrease in vaginal sensation after surgery d. Symptoms caused by the drop in estrogen level

A Venous thromboembolism (VTE) is a potential complication after the surgery, and the nurse will instruct the patient about ways to prevent it. Vaginal sensation is decreased after a vaginal hysterectomy but not after abdominal hysterectomy. Leiomyomas are benign tumors, so chemotherapy and radiation will not be prescribed. Because the patient will still have her ovaries, the estrogen level will not decrease. DIF: Cognitive Level: Apply (application) REF: 1298 TOP: Nursing Process: Implementation MSC:

A 28-year-old pregnant woman is spilling sugar in her urine. The physician orders a glucose tolerance test, which reveals gestational diabetes. The patient is shocked by the diagnosis, stating that she is conscientious about her health, and asks the nurse what causes gestational diabetes. The nurse should explain that gestational diabetes is a result of what etiologic factor? A) Increased caloric intake during the first trimester B) Changes in osmolality and fluid balance C) The effects of hormonal changes during pregnancy D) Overconsumption of carbohydrates during the first two trimesters

A 28-year-old pregnant woman is spilling sugar in her urine. The physician orders a glucose tolerance test, which reveals gestational diabetes. The patient is shocked by the diagnosis, stating that she is conscientious about her health, and asks the nurse what causes gestational diabetes. The nurse should explain that gestational diabetes is a result of what etiologic factor? A) Increased caloric intake during the first trimester B) Changes in osmolality and fluid balance C) The effects of hormonal changes during pregnancy D) Overconsumption of carbohydrates during the first two trimesters

Which nonhormonal therapies will the nurse suggest for a healthy perimenopausal woman who prefers not to use hormone therapy (HT) (select all that apply)? a. Reduce coffee intake. b. Exercise several times a week. c. Take black cohosh supplements. d. Have a glass of wine in the evening. e. Increase intake of dietary soy products.

A, B, C, E Reduction in caffeine intake, use of black cohosh, increasing dietary soy intake, and exercising three to four times weekly are recommended to reduce symptoms associated with menopause. Alcohol intake in the evening may increase the sleep problems associated with menopause. DIF: Cognitive Level: Analyze (analysis) REF: 1285 TOP: Nursing Process: Implementation MSC:

A patient who has stage IIB breast cancer has a left mastectomy with axillary node dissection. Ten lymph nodes are resected and three contain malignant cells. What is teh best goal for the nurse to use in planning care? A Assess the pt's response to pathology report B. Maintain joint flexibility and l. arm function B. Prevent L. arm lymphatic fluid accumulation D. Prepare her for chemo and radiation therapy

A. Because the nurse acts as the pt. advocate and because this time is an extremely stressful period for the patient and family, the nurses initial plan focuses on the patients response to the pathology report as part of a comprhensive patient assessment.

What should the nurse include in patient teaching when preparing a 45 year old patient for a routine mammography? A. Additional xrays can be needed for dense breast tissue B. Ultrasound confirms the presence of nonvisible mass C. Early detection of malignany tumore prevents mettastasis D. Continue to obtain clinical breast examinations every 3 years

A. Dense breast tissue on mammography can appear as a suspicious region of the breast. Once identified, additional mammograms can target the region for further examination

Two years after breast conserving surgery for a malignany left breast tumor, the nurse assesses the patient. Which patient assessment data is a priority? A. Irregular, fixed mass i the left axilla B. Temperature, HR, lung sounds C. Left upper extremity persistent edema D. Keloid formation around wound edges

A. The most frequent site of breast cancer recurrence is the site of the original tumor, so the presence of an irregular and fixed mass in the L. axilla is the nurse priority. Fixed and irregular are characteristics of a malignant breast tumor and require follow up care

The nurse is caring for a patient with chlamydial infection. For which complications does the nurse monitor this patient on the basis of the current diagnosis? Select all that apply. A. Abscess B. Infertility C. Cold sores D. Atonic bladder E. Reactive arthritis

A. Abscess B. Infertility E. Reactive arthritis

A patient is admitted to the hospital with severe pain due to a tuboovarian abscess caused by pelvic inflammatory disease (PID). Which nursing interventions are appropriate for the patient? Select all that apply. A. Administer IV fluids. B. Give a sitz bath to the patient. C. Provide bed rest in a prone position. D. Administer analgesics to relieve pain. E. Apply a cold compress to the lower abdomen.

A. Administer IV fluids. B. Give a sitz bath to the patient. D. Administer analgesics to relieve pain.

A couple who have experienced a spontaneous abortion before eight weeks of gestation visit a nurse for information about increasing their chances for successful future pregnancies. What information should the nurse provide? Select all that apply. A. Advise the couple that any bleeding should be reported immediately. B. Inform the couple that the abortion may have been due to chromosomal defects. C. Inform the couple that the abortion may have been due to cervical incompetence. D. Advise the couple that serial monitoring of serum follicular stimulating hormone (FSH) levels should be done. E. Advise the couple that absolute bed rest and abstention from intercourse can reduce the chance of spontaneous abortion.

A. Advise the couple that any bleeding should be reported immediately. B. Inform the couple that the abortion may have been due to chromosomal defects.

A patient is scheduled to have a hysterectomy and informs the nurse that after previous abdominal surgery, she had gas pains. What can the nurse educate the patient about that will reduce the occurrence of gas pains? A. Ambulating after surgery B. Performing leg exercises C. Changing position frequently D. Avoiding pressure under knees

A. Ambulating after surgery

A patient informs the nurse that she has not menstruated in several months. How will the nurse document this finding? A. Amenorrhea B. Dysmenorrhea C. Ectopic pregnancy D. Heavy menstrual bleeding

A. Amenorrhea

A patient reports tingling, burning, and itching in the genital area. Based on these symptoms, what type of medication does the nurse anticipate administering? A. Antivirals B. Antibiotics C. Vaccination D. Contraceptives

A. Antivirals

A nurse is caring for a patient who has premenstrual syndrome (PMS). The nurse should assess the patient for which symptoms? Select all that apply. A. Anxiety B. Hair loss C. Back pain D. Headaches E. Breast discomfort F. Abdominal bloating

A. Anxiety C. Back pain D. Headaches E. Breast discomfort F. Abdominal bloating

The nurse is caring for a patient that underwent colporrhaphy. Which nursing intervention would provide the most comfort to this patient? Select all that apply. A. Apply ice locally in the area of swelling. B. Provide sitz bath regularly to the patient. C. Administer cleansing enema in the morning. D. Apply vaginal cream after shaving perineal area. E. Ask the patient to perform perineal care twice daily.

A. Apply ice locally in the area of swelling. B. Provide sitz bath regularly to the patient. E. Ask the patient to perform perineal care twice daily.

What instructions should the nurse include when teaching a patient about ways to prevent reinfection by gonorrhea? Select all that apply. A. Avoid alcohol. B. Receive a vaccine to prevent gonorrhea. C. Abstain from sexual intercourse until the treatment is complete. D. Avoid going out in public and minimize physical contact with people. E. Avoid squeezing the penis to look for a discharge.

A. Avoid alcohol. C. Abstain from sexual intercourse until the treatment is complete. E. Avoid squeezing the penis to look for a discharge.

A patient is being discharged from the hospital after having a hysterectomy for a malignant uterine growth performed vaginally or via an incision. What instruction should the nurse give to the patient for rehabilitation? Select all that apply. A. Avoid dancing for several months. B. Avoid swimming for several months. C. Avoid lifting heavy weights for two months. D. Avoid sex for six days after the wound has healed. E. Avoid sex for six weeks after the wound has healed.

A. Avoid dancing for several months. C. Avoid lifting heavy weights for two months.

A patient is suspected of having a sexually transmitted disease. What are the possible routes through which the patient may have contracted the infection? Select all that apply. A. Blood transfusion B. Sharing of intravenous needles or syringes C. Consuming contaminated water D. Inhaling air in contact with an affected person E. Sexual contact with any person with a sexually transmitted infection (STI)

A. Blood transfusion B. Sharing of intravenous needles or syringes E. Sexual contact with any person with a sexually transmitted infection (STI)

A patient is reporting various symptoms of premenstrual syndrome (PMS). Which medication does the nurse anticipate discussing with the patient for the relief of these symptoms? A. Buspirone for anxiety B. Vitamin B 6 for cramping pain C. Spironolactone for negative mood D. Ibuprofen for premenstrual dysphoric disorder

A. Buspirone for anxiety

A patient comes to the clinic after being informed by a sexual partner of possible recent exposure to syphilis. The nurse will examine the patient for what characteristic finding of syphilis in the primary clinical stage? A. Chancre B. Alopecia C. Condylomata lata D. Regional adenopathy

A. Chancre

A patient reports abdominal pain, urinary urgency, and difficulty eating, and has a positive CA-125 tumor test. Which medications does the nurse anticipate administering? A. Cisplatin and paclitaxel B. Docetaxel and metformin C. Carboplatin and tamoxifen D. Altretamine and progesterone HT

A. Cisplatin and paclitaxel

A female college student presents at the campus clinic reporting a headache and increased lethargy with difficulty concentrating on studying. Which of the following questions should the nurse include during the focused assessment? Select all that apply. A. Does the patient practice birth control? B. When was the patient's last self-breast exam? C. Does the patient wear contact lenses or glasses? D. Does the patient have a history of menstrual cramps? E. Did the patient bump or receive any hits to her head?

A. Does the patient practice birth control? C. Does the patient wear contact lenses or glasses? E. Did the patient bump or receive any hits to her head?

The nurse is examining the diagnostic reports of a patient who has had a spontaneous abortion. The ultrasound reports indicate the presence of products of conception in the uterus. Which procedure does the primary health care provider perform? A. Hysterotomy B. Dilation and curettage C. Hypertonic saline solution D. Methotrexate with misoprostol

B. Dilation and curettage

The nurse is caring for a patient with genital herpes. What should the nurse emphasize while providing psychologic support to this patient? Select all that apply. A. Encourage the patient to verbalize feelings. B. Instruct the patient to avoid interaction with others. C. Help the patient realize that he or she should never be sexually active henceforth. D. Help the patient understand that the treatment could run for a long duration. E. Help the patient identify and avoid the factors that precipitated the condition.

A. Encourage the patient to verbalize feelings. D. Help the patient understand that the treatment could run for a long duration. E. Help the patient identify and avoid the factors that precipitated the condition.

A nurse is administering a human papillomavirus vaccine to an adolescent. What precaution does the nurse need to take while administering this vaccine? A. Ensure that the patient is lying down or sitting during vaccine administration. B. Ensure that the patient has an intravenous line placed before vaccine administration. C. Ensure that the patient has a complete liver profile done before receiving the vaccine. D. Ensure that the patient takes the vaccine on an empty stomach.

A. Ensure that the patient is lying down or sitting during vaccine administration.

Normal vaginal pH is typically lower than 4.5 as a result of sufficient levels of which combined substances? A. Estrogen and lactobacillus B. Estrogen and progesterone C. Progesterone and lactobacillus D. Testosterone and lactobacillus

A. Estrogen and lactobacillus

When performing an assessment of a patient with herpes simplex virus (HSV) type 1, which body parts would the nurse find to be affected? Select all that apply. A. Gingiva B. Upper respiratory tract C. Central nervous system D. Genital tract E. Perineum

A. Gingiva B. Upper respiratory tract C. Central nervous system

A patient is receiving treatment for a sexually transmitted infection prior to culture results being obtained. The nurse is to administer ceftriaxone intramuscularly. What sexually transmitted infection does the nurse recognize is being treated? A. Gonorrhea B. Candidiasis C. Herpes simplex virus type 2 D. Human immunodeficiency virus

A. Gonorrhea

A pregnant patient with abdominal pain and irregular vaginal bleeding has had morning sickness, breast tenderness, and syncope since the sixth week of pregnancy. For which complication should the nurse monitor the patient? Select all that apply. A. Hemorrhage B. Peripheral edema C. Migraine headache D. Hypovolemic shock E. Thyroid dysfunction

A. Hemorrhage D. Hypovolemic shock

A patient suspected of having syphilis tests positive on the Venereal Disease Research Laboratory (VDRL) test. Which other diseases can give false-positive results when testing for syphilis? Select all that apply. A. Hepatitis B. Hyperlipidemia C. Infectious mononucleosis D. Systemic lupus erythematosus E. Type 2 diabetes mellitus

A. Hepatitis C. Infectious mononucleosis D. Systemic lupus erythematosus

Which of the following factors are poor prognostic indicators of breast cancer (select all that apply)? A. Previous breast cancer B. Well-differentiated tumor C. High incidence of aneuploidy D. Estrogen and progesterone receptor positive E. Many cells in synthesis phase of cell cycle

A. History of previous breast cancer increases the risk of recurrence. Tumors that are well differentiated tend to be less affressive than undifferentiated tumors. Tumors unreponsive to hormonal therapy tend to be more aggressive

When teaching a group of postmenopausal women about the risk of osteoporosis, what information should the nurse include? Select all that apply. A. Inform that raloxifene prevents bone loss. B. Emphasize the need for estrogen replacement. C. Inform that Bisphosphonates enhance bone mineral density. D. Emphasize the need for adequate intake of calcium and vitamin E. E. Encourage avoidance of weight-bearing exercises, because they can cause bone loss.

A. Inform that raloxifene prevents bone loss. B. Emphasize the need for estrogen replacement. C. Inform that Bisphosphonates enhance bone mineral density.

Which nonhormonal measures should the nurse teach to a postmenopausal patient experiencing hot flashes to decrease heat production and increase heat loss? Select all that apply. A. Limit intake of alcohol. B. Increase intake of caffeine. C. Use heavy quilts when sleeping. D. Promote air circulation in the room. E. Avoid wearing clothes with tight necks.

A. Limit intake of alcohol. D. Promote air circulation in the room. E. Avoid wearing clothes with tight necks.

For which treatment does the nurse prepare the patient with leiomyomas whose pelvic examination reveals uterine fibroids? Select all that apply. A. Myomectomy B. Hysteroscope C. Laser resection instruments D. Uterine artery embolization E. Magnetic resonance imaging (MRI)-guided focused ultrasound

A. Myomectomy C. Laser resection instruments E. Magnetic resonance imaging (MRI)-guided focused ultrasound

The nurse is performing an assessment for a patient with secondary dysmenorrhea. Which findings support the patient's diagnosis? Select all that apply. A. Nausea B. Fatigue C. Headache D. Weight gain E. Constipation F. Painful menses

A. Nausea B. Fatigue C. Headache F. Painful menses

The nurse is assessing a patient suspected of having a chlamydial infection. What diagnostic tests should the nurse prepare the patient for? Select all that apply. A. Nucleic acid amplification test B. Direct fluorescent antibody test C. Venereal Disease Research Laboratory test D. Enzyme immunoassay test E. Western blot test

A. Nucleic acid amplification test B. Direct fluorescent antibody test D. Enzyme immunoassay test

The nurse is caring for a patient who has polycystic ovary syndrome (PCOS). When preparing to teach this patient, the nurse should know that which classic manifestation is associated with the severity of symptoms, including infertility? A. Obesity B. Hirsutism C. Amenorrhea D. Irregular menstrual periods

A. Obesity

A newly married couple is interested in using a combined oral contraceptive containing both estrogen and progesterone. What information should the nurse include when explaining about combined oral contraceptives to this couple? Select all that apply. A. Oral contraceptives increase risk of breast cancer. B. Oral contraceptives increase risk of ovarian cancer. C. Oral contraceptives decrease the risk of liver cancer. D. Oral contraceptives need to be taken after sexual intercourse. E. Oral contraceptives increase blood pressure and serum cholesterol.

A. Oral contraceptives increase risk of breast cancer. E. Oral contraceptives increase blood pressure and serum cholesterol.

When performing a physical examination of a male patient with suspected gonorrhea, what symptoms is the nurse likely to find? Select all that apply. A. Pain during urination B. Purulent discharge from the urethra C. Warts and growths on the penis D. Swollen testicles E. Erectile dysfunction

A. Pain during urination B. Purulent discharge from the urethra D. Swollen testicles

A patient with neurosyphilis is being discharged after treatment. What follow-up care should the nurse advise for the patient? Select all that apply. A. Periodic serologic testing B. Clinical evaluation at six-month intervals C. Periodic cerebrospinal fluid examinations for at least three years D. Continued use of oral acyclovir E. Viral cultures of active lesions

A. Periodic serologic testing B. Clinical evaluation at six-month intervals C. Periodic cerebrospinal fluid examinations for at least three years

A female patient is admitted to the nursing unit with dehydration related to frequent nausea and emesis. She was seen in the emergency department (ED) three weeks ago for a concussion and discharged home with instructions for physical and cognitive rest. Patient education and preparation should be done for which of the following tests? A. Pregnancy test B. Incentive spirometry C. 12-lead electrocardiogram (EKG) D. Magnetic resonance imaging (MRI) scan of the brain

A. Pregnancy test

A patient has been admitted for dilation and evacuation (D&E). What nursing interventions should the nurse perform for this patient? A. Prepare the patient both physically and emotionally for the procedure. B. Counsel the patient that she may fail to conceive in the future due to this procedure. C. Instruct the patient that postprocedure she has to avoid sexual intercourse for two months. D. Inform the patient that this procedure is associated with the possibility of multiple pregnancies in the future.

A. Prepare the patient both physically and emotionally for the procedure.

After assessing a pregnant patient, the nurse anticipates that the patient has a risk of spontaneous abortion. Which advice given by the nurse is beneficial for the patient? A. "You should move frequently." B. "You should avoid intercourse." C. "You should avoid high-fiber foods." D. "You should increase folic acid supplements."

B. "You should avoid intercourse."

The nurse is teaching a group of parents at a local high school about the importance of the human papilloma virus vaccine. Which strategies by the nurse would help address religious concerns? Select all that apply. A. Providing accurate and current information about the vaccine B. Discussing the prevention of cancer as the major role of vaccine C. Discussing why religious concerns are not important enough to prevent use of the vaccine D. Allowing parents to ask questions and express concerns about the vaccine in an open, supportive environment E. Informing parents that they will be responsible for their children's death from a preventable cancer if they do not vaccinate

A. Providing accurate and current information about the vaccine B. Discussing the prevention of cancer as the major role of vaccine D. Allowing parents to ask questions and express concerns about the vaccine in an open, supportive environment

A female patient was admitted to the hospital after her mother found her sobbing in the bathroom, washing blood from her clothing and bandaging a laceration from a split eyebrow. Which health care team member would the nurse anticipate the health care provider to consult? Select all that apply. A. Psychiatrist B. Social worker C. Physical therapist D. Occupational therapist E. Speech and hearing specialist F. Sexual assault nurse examiner (SANE)

A. Psychiatrist B. Social worker F. Sexual assault nurse examiner (SANE)

The nurse is educating a group of adolescent girls about cervical cancer. What should the nurse include in the discussion? Select all that apply. A. Smoking is a risk factor for cervical cancer. B. Limiting sexual activity is recommended during adolescence. C. Taking oral contraceptive pills lowers the risk of cervical cancer. D. Doing regular Pap tests is important to screen for cervical cancer. E. Using condoms provides protection if they have multiple sexual partners.

A. Smoking is a risk factor for cervical cancer. B. Limiting sexual activity is recommended during adolescence. D. Doing regular Pap tests is important to screen for cervical cancer.

The nurse working in the emergency department has seen several patients with sexually transmitted infections (STIs) that must be reported to state or local health departments. Which STIs must the nurse report? Select all that apply. A. Syphilis B. Gonorrhea C. Herpes simplex virus (HSV) D. Human papilloma virus (HPV) E. Condylomata acuminate

A. Syphilis B. Gonorrhea

A parent is deciding whether to allow an adolescent child to receive the Gardasil vaccine. What information should the nurse provide to the parent? Select all that apply. A. The vaccine provides protection against certain types of cancer. B. Often persons with human papillomavirus (HPV) are asymptomatic. C. The vaccine protects against HPV types 6, 11, 16, 18. D. One dose of the vaccine protects against HPV. E. The vaccine is used to treat active infections. F. It is recommended because one does not know if one's child is sexually active.

A. The vaccine provides protection against certain types of cancer. B. Often persons with human papillomavirus (HPV) are asymptomatic. C. The vaccine protects against HPV types 6, 11, 16, 18.

Human chorionic gonadotropin is prescribed to an infertile patient. What explanation will the nurse provide when the patient asks why they will be taking this? A. To induce ovulation B. To promote the development of follicles C. To suppress the release of follicle-stimulating hormone D. To prevent the premature release of luteinizing hormone

A. To induce ovulation

The nurse is assisting the health care provider with a routine vaginal exam on a female patient. The patient has erythema of the vulva, and during the breast exam, scratches were noted. The patient is withdrawn and reluctant to answer many questions about her health history except to volunteer she is tired after attending a party the previous night. Which of the following is acceptable without patient consent? A. Using an unlubricated speculum B. Sending a cervical culture for gonorrhea C. Clipping a pubic hair sample to send to the lab D. Sending a blood specimen for a pregnancy test

A. Using an unlubricated speculum

A patient newly diagnosed with leiomyomas becomes very anxious regarding the diagnosis. What information should the nurse give to this patient to alleviate anxiety? Select all that apply. A. Uterine fibroids may result in abortion or infertility. B. Uterine fibroids are benign tumors with a good prognosis. C. Uterine fibroids are slow-growing malignant tumors with a good prognosis. D. Uterine fibroids are benign tumors but may undergo malignant transformation. E. Uterine fibroids are slow-growing benign tumors that undergo atrophy after menopause.

A. Uterine fibroids may result in abortion or infertility. B. Uterine fibroids are benign tumors with a good prognosis. E. Uterine fibroids are slow-growing benign tumors that undergo atrophy after menopause.

A 70-year-old patient who has had a transurethral resection of the prostate (TURP) for benign prostatic hyperplasia (BPH) is being discharged from the hospital today, The nurse determines that additional instruction is needed when the patient says which of the following? a. "I should call the doctor if I have incontinence at home." b. "I will avoid driving until I get approval from my doctor." c. "I will increase fiber and fluids in my diet to prevent constipation." d. "I should continue to schedule yearly appointments for prostate exams."

ANS: A Because incontinence is common for several weeks after a TURP, the patient does not need to call the health care provider if this occurs. The other patient statements indicate that the patient has a good understanding of post-TURP instructions

A 24-year-old female says she wants to begin using oral contraceptives. Which information from the nursing assessment is most important to report to the health care provider? a. The patient quit smoking 5 months previously. b. The patient's blood pressure is 154/86 mm Hg. c. The patient has not been vaccinated for rubella. d. The patient has chronic iron-deficiency anemia.

ANS: B Because hypertension increases the risk for morbidity and mortality in women taking oral contraceptives, the patient's blood pressure should be controlled before oral contraceptives are prescribed. The other information also will be reported but will not affect the choice of contraceptive.

A 58-year-old man with erectile dysfunction (ED) tells the nurse he is interested in using sildenafil (Viagra). Which action should the nurse take first? a. Assure the patient that ED is common with aging. b. Ask the patient about any prescription drugs he is taking. c. Tell the patient that Viagra does not always work for ED. d. Discuss the common adverse effects of erectogenic drugs.

ANS: B Because some medications can cause ED and patients using nitrates should not take sildenafil, the nurse should first assess for prescription drug use. The nurse may want to teach the patient about realistic expectations and adverse effects of sildenafil therapy, but this should not be the first action. Although ED does increase with aging, it may be secondary to medication use or cardiovascular disease

A 58-year-old patient who has been recently diagnosed with benign prostatic hyperplasia (BPH) tells the nurse that he does not want to have a transurethral resection of the prostate (TURP) because it might affect his ability to maintain an erection during intercourse. Which action should the nurse take? a. Provide teaching about medications for erectile dysfunction (ED). b. Discuss that TURP does not commonly affect erectile function. c. Offer reassurance that sperm production is not affected by TURP. d. Discuss alternative methods of sexual expression besides intercourse.

ANS: B ED is not a concern with TURP, although retrograde ejaculation is likely and the nurse should discuss this with the patient. Erectile function is not usually affected by a TURP, so the patient will not need information about penile implants or reassurance that other forms of sexual expression may be used. Because the patient has not asked about fertility, reassurance about sperm production does not address his concerns

A 22-year-old patient reports her concern about not having a menstrual period for the past 7 months. Which statement by the patient indicates a possible related factor to the amenorrhea? a. "I drink at least 3 glasses of nonfat milk every day." b. "I run 7 to 8 miles every day to keep my weight down." c. "I was treated for a sexually transmitted infection 2 years ago." d. "I am not sexually active but currently I have an IUD."

ANS: B Excessive exercise can cause amenorrhea. The other statements by the patient do not suggest any urgent teaching needs

A 22-year-old patient reports her concern about not having a menstrual period for the past 7 months. Which statement by the patient indicates a possible related factor to the amenorrhea? a. "I drink at least 3 glasses of nonfat milk every day." b. "I run 7 to 8 miles every day to keep my weight down." c. "I was treated for a sexually transmitted infection 2 years ago." d. "I am not sexually active but currently I have an IUD."

ANS: B Excessive exercise can cause amenorrhea. The other statements by the patient do not suggest any urgent teaching needs.

A patient admitted with chest pain is also found to have positive Venereal Disease Research Laboratory (VDRL) and fluorescent treponemal antibody absorption (FAT-Abs) tests, rashes on the palms and the soles of the feet, and moist papules in the anal and vulvar area. Which action will the nurse include in the plan of care? a. Assess for arterial aneurysms. b. Wear gloves for patient contact. c. Place the patient in a private room. d. Apply antibiotic ointment to the perineum.

ANS: B Exudate from any lesions with syphilis is highly contagious. Systemic antibiotics, rather than local treatment of lesions, are used to treat syphilis. The patient does not require a private room because the disease is spread through contact with the lesions. This patient has clinical manifestations of secondary syphilis and does not need to be monitored for manifestations of tertiary syphilis

A couple is scheduled to have a Huhner test for infertility. In preparation for the test, the nurse will instruct the couple about a. being sedated during the procedure. b. determining the estimated time of ovulation. c. experiencing shoulder pain after the procedure. d. refraining from intercourse before the appointment.

ANS: B For the Huhner test, the couple should have intercourse at the estimated time of ovulation and then arrive for the test 2 to 8 hours after intercourse. The other instructions would be used for other types of fertility testing

A 48-year-old male patient who has been diagnosed with gonococcal urethritis tells the nurse he had recent sexual contact with a woman but says she did not appear to have any disease. In responding to the patient, the nurse explains that a. women do not develop gonorrhea infections but can serve as carriers to spread the disease to males. b. women may not be aware they have gonorrhea because they often do not have symptoms of infection. c. women develop subclinical cases of gonorrhea that do not cause tissue damage or clinical manifestations. d. when gonorrhea infections occur in women, the disease affects only the ovaries and not the genital organs.

ANS: B Many women with gonorrhea are asymptomatic or have minor symptoms that are overlooked. The disease may affect both the genitals and the other reproductive organs and cause complications such as pelvic inflammatory disease (PID). Women who can transmit the disease have active infections

The nurse performing a focused examination to determine possible causes of infertility will assess for a. hydrocele. b. varicocele. c. epididymitis. d. paraphimosis.

ANS: B Persistent varicoceles are commonly associated with infertility. Hydrocele, epididymitis, and paraphimosis are not risk factors for infertility

A 27-year-old man who has testicular cancer is being admitted for a unilateral orchiectomy. The patient does not talk to his wife and speaks to the nurse only to answer the admission questions. Which action is best for the nurse to take? a. Teach the patient and the wife that impotence is unlikely after unilateral orchiectomy. b. Ask the patient if he has any questions or concerns about the diagnosis and treatment. c. Document the patient's lack of communication on the chart and continue preoperative care. d. Inform the patient's wife that concerns about sexual function are common with this diagnosis.

ANS: B The initial action by the nurse should be assessment for any anxiety or questions about the surgery or postoperative care. The nurse should address the patient, not the spouse, when discussing the diagnosis and any possible concerns. Without further assessment of patient concerns, the nurse should not offer teaching about complications after orchiectomy. Documentation of the patient's lack of interaction is not an adequate nursing action in this situation

The nurse will plan to teach the patient scheduled for photovaporization of the prostate (PVP) a. that urine will appear bloody for several days. b. how to care for an indwelling urinary catheter. c. that symptom improvement takes 2 to 3 weeks. d. about complications associated with urethral stenting.

ANS: B The patient will have an indwelling catheter for 24 to 48 hours and will need teaching about catheter care. There is minimal bleeding with this procedure. Symptom improvement is almost immediate after PVP. Stent placement is not included in the procedure

A 52-year-old man tells the nurse that he decided to seek treatment for erectile dysfunction (ED) because his wife "is losing patience with the situation." The most appropriate nursing diagnosis for the patient is a. situational low self-esteem related to effects of ED. b. ineffective role performance related to effects of ED. c. anxiety related to inability to have sexual intercourse. d. ineffective sexuality patterns related to infrequent intercourse.

ANS: B The patient's statement indicates that the relationship with his wife is his primary concern. Although anxiety, low self-esteem, and ineffective sexuality patterns may also be concerns, the patient information suggests that addressing the role performance problem will lead to the best outcome for this patient

Which topics will the nurse include when preparing to teach a patient with recurrent genital herpes simplex (select all that apply)? a. Infected areas should be kept moist to speed healing. b. Sitz baths may be used to relieve discomfort caused by the lesions. c. Genital herpes can be cured by consistent use of antiviral medications. d. Recurrent genital herpes episodes usually are shorter than the first episode. e. The virus can infect sexual partners even when you do not have symptoms of infection.

ANS: B, D, E Patients are taught that shedding of the virus and infection of sexual partners can occur even in asymptomatic periods, that recurrent episodes resolve more quickly, and that sitz baths can be used to relieve pain caused by the lesions. Antiviral medications decrease the number of outbreaks, but do not cure herpes simplex infections. Infected areas may be kept dry if this decreases pain and itching

A 19-year-old patient calls the school clinic and tells the nurse, "My menstrual period is very heavy this time. I have to change my tampon every 4 hours." Which action should the nurse take next? a. Tell the patient that her flow is not unusually heavy. b. Schedule the patient for an appointment later that day. c. Ask the patient how heavy her usual menstrual flow is. d. Have the patient call again if the heavy flow continues.

ANS: C Because a heavy menstrual flow is usually indicated by saturating a pad or tampon in 1 to 2 hours, the nurse should first assess how heavy the patient's usual flow is. There is no need to schedule the patient for an appointment that day. The patient may need to call again, but this is not the first action that the nurse should take. Telling the patient that she does not have a heavy flow implies that the patient's concern is not important

Which information will the nurse teach a patient who has chronic prostatitis? a. Ibuprofen (Motrin) should provide good pain control. b. Prescribed antibiotics should be taken for 7 to 10 days. c. Intercourse or masturbation will help relieve symptoms. d. Cold packs used every 4 hours will decrease inflammation.

ANS: C Ejaculation helps drain the prostate and relieve pain. Warm baths are recommended to reduce pain. Nonsteroidal antiinflammatory drugs (NSAIDs) are frequently prescribed but usually do not offer adequate pain relief. Antibiotics for chronic prostatitis are taken for 4 to 12 weeks

Which action by the unlicensed assistive personnel (UAP) who are assisting with the care of patients with male reproductive problems indicates that the nurse should provide more teaching? a. The UAP apply a cold pack to the scrotum for a patient with mumps orchitis. b. The UAP help a patient who has had a prostatectomy to put on antiembolism hose. c. The UAP leave the foreskin pulled back after cleaning the glans of a patient who has a retention catheter. d. The UAP encourage a high oral fluid intake for patient who had transurethral resection of the prostate yesterday.

ANS: C Paraphimosis can be caused by failing to replace the foreskin back over the glans after cleaning. The other actions by UAP are appropriate

A 49-year-old man who has type 2 diabetes, high blood pressure, hyperlipidemia, and gastroesophageal reflux tells the nurse that he has had recent difficulty in achieving an erection. Which of the following drugs from his current medications list may cause erectile dysfunction (ED)? a. Ranitidine (Zantac) b. Atorvastatin (Lipitor) c. Propranolol (Inderal) d. Metformin (Glucophage)

ANS: C Some antihypertensives may cause erectile dysfunction, and the nurse should anticipate a change in antihypertensive therapy. The other medications will not affect erectile function.

When obtaining the pertinent health history for a man who is being evaluated for infertility, which question is most important for the nurse to ask? a. "Are you circumcised?" b. "Have you had surgery for phimosis?" c. "Do you use medications to improve muscle mass?" d. "Is there a history of prostate cancer in your family?"

ANS: C Testosterone or testosterone-like medications may adversely affect sperm count. The other information will be obtained in the health history but does not affect the patient's fertility.

The following patients call the outpatient clinic. Which phone call should the nurse return first? a. A 44-year-old patient who has bloody discharge after a hysteroscopy earlier today b. A 64-year-old patient who is experiencing shoulder pain after a laparoscopy yesterday c. A 34-year-old patient who is short of breath after pelvic computed tomography (CT) with contrast d. A 54-year-old patient who has severe breast tenderness following a needle aspiration breast biopsy

ANS: C The patient's dyspnea suggests a delayed reaction to the iodine dye used for the CT scan. The other patient's symptoms are not unusual after the procedures they had done.

Which statement by a 24-year-old patient indicates that the nurse's teaching about management of primary genital herpes has been effective? a. "I will use acyclovir ointment on the area to relieve the pain." b. "I will use condoms for intercourse until the medication is all gone." c. "I will take the acyclovir (Zovirax) every 8 hours for the next week." d. "I will need to take all of the medication to be sure the infection is cured."

ANS: C The treatment regimen for primary genital herpes infections includes acyclovir 400 mg 3 times daily for 7 to 10 days. The patient is taught to abstain from intercourse until the lesions are gone. (Condoms should be used even when the patient is asymptomatic.) Acyclovir ointment is not effective in treating lesions or reducing pain. Herpes infection is chronic and recurrent

After the nurse has taught a patient with a newly diagnosed sexually transmitted infection about expedited partner therapy, which patient statement indicates that the teaching has been effective? a. "I will tell my partner that it is important to be examined at the clinic." b. "I will have my partner take the antibiotics if any STI symptoms occur." c. "I will make sure that my partner takes all of the prescribed medication." d. "I will have my partner use a condom until I have finished the antibiotics."

ANS: C With expedited partner therapy, the patient is given a prescription or medications for the partner. The partner does not need to be evaluated by the health care provider, but is presumed to be infected and should be treated concurrently with the patient. Use of a condom will not treat the presumed STI in the partner

A 55-year-old woman in the sexually transmitted infection (STI) clinic tells the nurse that she is concerned she may have been exposed to gonorrhea by her partner. To determine whether the patient has gonorrhea, the nurse will plan to a. interview the patient about symptoms of gonorrhea. b. take a sample of cervical discharge for Gram staining. c. draw a blood specimen or rapid plasma reagin (RPR) testing. d. obtain secretions for a nucleic acid amplification test (NAAT).

ANS: D NAAT has a high sensitivity (similar to a culture) for gonorrhea. Because women have few symptoms of gonorrhea, asking the patient about symptoms may not be helpful in making a diagnosis. Smears and Gram staining are not useful because the female genitourinary tract has many normal flora that resemble N. gonorrhoeae. RPR testing is used to detect syphilis

After a 26-year-old patient has been treated for pelvic inflammatory disease, the nurse will plan to teach about a. use of hormone therapy (HT). b. irregularities in the menstrual cycle. c. changes in secondary sex characteristics. d. possible difficulty with becoming pregnant.

ANS: D Pelvic inflammatory disease may cause scarring of the fallopian tubes and result in difficulty in fertilization or implantation of the fertilized egg. Because ovarian function is not affected, the patient will not require HT, have irregular menstrual cycles, or experience changes in secondary sex characteristics.

To determine the severity of the symptoms for a 68-year-old patient with benign prostatic hyperplasia (BPH) the nurse will ask the patient about a. blood in the urine. b. lower back or hip pain. c. erectile dysfunction (ED). d. force of the urinary stream.

ANS: D The American Urological Association (AUA) Symptom Index for a patient with BPH asks questions about the force and frequency of urination, nocturia, etc. Blood in the urine, ED, and back or hip pain are not typical symptoms of BPH

A 68-year-old male patient tells the nurse that he is worried because he does not respond to sexual stimulation the same way he did when he was younger. The nurse's best response to the patient's concern is which of the following? a. "Interest in sex frequently decreases as men get older." b. "Many men need additional sexual stimulation with aging." c. "Erectile dysfunction is a common problem with older men." d. "Tell me more about how your sexual response has changed."

ANS: D The initial response by the nurse should be further assessment of the problem. The other statements by the nurse are accurate but may not respond to the patient's concerns

The newborn nursery nurse administers Erythromycin ophthalmic ointment prophylactically to a neonate to prevent blindness. What is the nurse explanation to the parent as to why this is being done? A. Due to potential exposure to syphilis B. Due to potential exposure to gonorrhea C. Due to potential exposure to chlamydia D. Due to potential exposure to Pseudomonas

B. Due to potential exposure to gonorrhea

Which of these nursing interventions for the patient who has had right-sided breast-conservation surgery and an axillary lymph node dissection is appropriate to assign to an LPN/LVN? a. Administering an analgesic 30 minutes before the scheduled arm exercises b. Teaching the patient how to avoid injury to the right arm c. Assessment of the patient's range of motion for the right arm d. Evaluation of the patient's understanding of discharge instructions about drain care

Answer: A Rationale: LPN/LVN education and scope of practice include administration and evaluation of the effects of analgesics. Assessment, teaching, and evaluation of a patient's understanding of instructions are more complex tasks that are more appropriate to RN-level education and scope of practice. Cognitive Level: Application Text Reference: pp. 1356-1361 Nursing Process: Implementation NCLEX: Safe and Effective Care Environment

A 38-year-old woman is scheduled for a breast-conservation therapy with a lumpectomy. As the nurse prepares her for surgery, she begins to cry and says, "I just do not know how to handle all of this." An appropriate response to the patient by the nurse is, a. "Would you like to talk about how you are feeling right now?" b. "I can see you are really upset. Would you like to be alone for a while?" c. "The important thing is that the tumor was found and is going to be removed." d. "With this surgery you will have very little change in the appearance of your breast."

Answer: A Rationale: The nurse encourages the patient to express feelings about the diagnosis and surgery. The response beginning, "I can see you are really upset" may indicate that the nurse is uncomfortable being with the patient while she is upset. The response beginning, "The important thing is that the tumor was found" places the nurse's value system above the patient's current concerns. And the response, "With this surgery you will have very little change in the appearance of your breast" does not address all the patient's possible concerns and is not true. Cognitive Level: Application Text Reference: p. 1360 Nursing Process: Implementation NCLEX: Psychosocial Integrity

After the nurse completes discharge teaching for a patient who has had a left modified radical mastectomy and lymph node dissection, which statement by the patient indicates that no further teaching is needed? a. "I will avoid reaching over the stove with my left hand." b. "I will need to do breast self-examination on my right breast monthly." c. "I will keep my left arm elevated until I go to bed." d. "I will remember to use my right arm and to rest the left one."

Answer: A Rationale: The patient should avoid any activity that might injure the left arm, such as reaching over a burner. Breast self-examination should be done to the right breast and the left mastectomy site. The left arm should be elevated when the patient is lying down also. The left arm should be used to improve range of motion and function. Cognitive Level: Application Text Reference: pp. 1359-1360 Nursing Process: Evaluation NCLEX: Physiological Integrity

A patient with premenstrual syndrome (PMS) approaches the nurse for advice on conservative ways of managing the condition. What should the nurse advise? Select all that apply. A. Drink coffee daily. B. Exercise on a regular basis. C. Practice relaxation techniques. D. Increase intake of refined carbohydrates. E. Increase consumption of pork, milk, and legumes.

B. Exercise on a regular basis. C. Practice relaxation techniques. E. Increase consumption of pork, milk, and legumes.

A patient with a small breast lump is advised to have a fine needle aspiration (FNA) biopsy. The nurse explains that an advantage to this procedure is that a. only a small incision is necessary, resulting in minimal breast pain and scarring. b. if the specimen is positive for malignancy, the patient can be told at the visit. c. if the specimen is negative for malignancy, the patient's fears of cancer can be put to rest. d. FNA is guided by a mammogram, ensuring that cells are taken from the lesion.

Answer: B Rationale: An FNA should only be done when an experienced cytologist is available to read the specimen immediately. If the specimen is positive for malignancy, the patient can be given this information immediately. No incision is needed. If the specimen is negative for malignancy, the patient will require biopsy of the lump. FNA is not guided by mammography. Cognitive Level: Application Text Reference: p. 1345 Nursing Process: Planning NCLEX: Physiological Integrity

A 51-year-old woman at menopause is considering the use of hormone replacement therapy (HRT) but is concerned about the risk of breast cancer. When discussing this issue with the patient, the nurse explains that a. HRT does not appear to increase the risk for breast cancer unless there are other risk factors. b. she and her health care provider must weigh the benefits of HRT against the possible risks of breast cancer. c. HRT is a safe therapy for menopausal symptoms if there is no family history of BRCA genes. d. alternative therapies with herbs and natural drugs are as effective as estrogen in relieving the symptoms of menopause.

Answer: B Rationale: Because HRT has been linked to increased risk for breast cancer, the patient and provider must determine whether or not to use HRT. Breast cancer incidence is increased in women using HRT, independent of other risk factors. HRT increase the risk for non-BRCA-associated cancer as well as for BRCA-related cancers. Alternative therapies can be used but are not consistent in relieving menopausal symptoms. Cognitive Level: Application Text Reference: p. 1348 Nursing Process: Implementation NCLEX: Physiological Integrity

At a routine health examination, a woman whose mother had breast cancer asks the nurse about the genetic basis of breast cancer and the genes involved. The nurse explains that a. her risk of inheriting BRCA gene mutations is small unless her mother had both ovarian and breast cancer. b. changes in BRCA genes that normally suppress cancer growth can be passed to offspring, increasing the risk for breast cancer. c. because her mother had breast cancer, she has inherited a 50% to 85% chance of developing breast cancer from mutated genes. d. genetic mutations increase cancer risk only in combination with other risk factors such as obesity.

Answer: B Rationale: Family history is a risk factor for breast cancer, and the nurse should discuss testing for BRCA genes with the patient. Although the BRCA gene is associated with increased risk for breast and ovarian cancer, the patient may be at risk if her mother had either one. About 5% to 10% of patients with breast cancer may have a genetic abnormality that contributes to breast cancer development. Risk factors are cumulative, but a family history alone will increase breast cancer risk. Cognitive Level: Application Text Reference: p. 1349 Nursing Process: Implementation NCLEX: Physiological Integrity

A 20-year-old student comes to the student health center after discovering a small painless lump in her right breast. She is worried that she might have cancer because her mother had cervical cancer. The nurse's response to the patient is based on the knowledge that the most likely cause of the breast lump is a. fibrocystic complex. b. fibroadenoma. c. breast abscess. d. adenocarcinoma.

Answer: B Rationale: Fibroadenoma is the most frequent cause of breast lumps in women under 25 years of age. Fibrocystic changes occur most frequently in women ages 35 to 50. Breast abscess is associated with pain and other systemic symptoms. Breast cancer is uncommon in women younger than 25. Cognitive Level: Application Text Reference: p. 1347 Nursing Process: Implementation NCLEX: Physiological Integrity

When teaching a 22-year-old patient about breast self-examination (BSE), the nurse will instruct the patient that a. BSE will reduce the risk of dying from breast cancer. b. performing BSE right after the menstrual period will improve comfort. c. BSE should be done daily while taking a bath or shower. d. annual mammograms should be scheduled in addition to BSE.

Answer: B Rationale: Performing BSE at the end of the menstrual period will reduce the breast tenderness associated with the procedure. The evidence is not clear that BSE reduces breast cancer mortality. BSE should be done monthly. Annual mammograms are not routinely scheduled for women under age 40. Cognitive Level: Application Text Reference: p. 1344 Nursing Process: Implementation NCLEX: Health Promotion and Maintenance

A patient has a permanent breast implant inserted in the outpatient surgery area. Which instructions will the nurse include in the discharge teaching? a. Resume normal activities 2 to 3 days after the mammoplasty. b. Check wound drains for excessive blood or any foul odor. c. Wear a loose-fitting bra to decrease irritation of the sutures. d. Take aspirin every 4 hours to reduce inflammation.

Answer: B Rationale: The patient should be taught drain care because the drains will be in place for 2 or 3 days after surgery. Normal activities can be resumed after 2 to 3 weeks. A bra that provides good support is typically ordered. Aspirin will decrease coagulation and is typically not given after surgery. Cognitive Level: Comprehension Text Reference: p. 1362 Nursing Process: Implementation NCLEX: Physiological Integrity

A 62-year-old patient complains to the nurse that mammograms are painful and a source of radiation exposure. She says she does breast self-examination (BSE) monthly and asks whether it is necessary to have an annual mammogram. The nurse's best response to the patient is, a. "If your mammogram was painful, it is especially important that you have it done annually." b. "An ultrasound examination of the breasts, which is not painful or a source of radiation, can be substituted for a mammogram." c. "Because of your age, it is even more important for you to have annual mammograms." d. "Unless you find a lump while examining your breasts, a mammogram every 2 years is recommended after age 60."

Answer: C Rationale: Annual mammograms are recommended for women over age 40 as long as they are in good health. The incidence of breast cancer increases in women over 60. Pain with a mammogram does not indicate any greater risk for breast cancer. Ultrasound may be used in some situations to differentiate cystic breast problems from cancer but is not a substitute for annual mammograms. Cognitive Level: Application Text Reference: p. 1344 Nursing Process: Implementation NCLEX: Health Promotion and Maintenance

The nurse administers mifepristone to the patient in the seventh week of pregnancy to induce abortion. Which medication would the nurse administer intravaginally to the patient after two days? A. Carboplast B. Misoprostol C. Methotrexate D. Dinoprostone

B. Misoprostol

A 33-year-old patient tells the nurse that she has fibrocystic breasts but reducing her sodium and caffeine intake and other measures have not made a difference in the fibrocystic condition. An appropriate patient outcome for the patient is a. calls the health care provider if any lumps are painful or tender. b. states the reason for immediate biopsy of new lumps. c. monitors changes in size and tenderness of all lumps in relation to her menstrual cycle. d. has genetic testing for BRCA-1 and BRCA-2 to determine her risk for breast cancer.

Answer: C Rationale: Because fibrocystic breasts may increase in size and tenderness during the premenstrual phase, the patient is taught to monitor for this change and to call if the changes persist after menstruation. Pain and tenderness are typical of fibrocystic breasts, and the patient should not call for these symptoms. New lumps may be need biopsy if they persist after the menstrual period, but the biopsy is not done immediately. The existence of fibrocystic breasts is not associated with the BRCA genes. Cognitive Level: Application Text Reference: pp. 1346-1347 Nursing Process: Planning NCLEX: Health Promotion and Maintenance

A patient at the clinic who has metastatic breast cancer has a new prescription for trastuzumab (Herceptin). The nurse will plan to a. teach the patient about the need to monitor serum electrolyte levels. b. ask the patient to call the health care provider before using any over-the-counter (OTC) pain relievers. c. instruct the patient to call if she notices ankle swelling. d. have the patient schedule frequent eye examinations.

Answer: C Rationale: Herceptin can lead to ventricular dysfunction, so the patient is taught to self-monitor for symptoms of heart failure. There is no need to monitor serum electrolyte levels. OTC pain relievers do not interact with Herceptin. Changes in visual acuity may occur with tamoxifen, but not with Herceptin. Cognitive Level: Analysis Text Reference: p. 1356 Nursing Process: Implementation NCLEX: Physiological Integrity

Following a modified radical mastectomy, a patient tells the nurse the health care provider has recommended a flap procedure for breast reconstruction but that she did not understand how this was done. The nurse explains that the most common procedure, a transverse rectus abdominis musculocutaneous (TRAM) flap, involves a. relocating muscle tissue from the back and using it to form a breast. b. removing a portion of an abdominal muscle to use as breast tissue. c. pulling part of the abdominal muscle up to the breast area through a tunnel in the chest. d. relocating the arteries from the abdominal muscle to improve circulation to the implant.

Answer: C Rationale: In the TRAM flap, part of the rectus abdominis muscle is tunneled to the breast area and molded to form a breast. In the latissimus dorsi musculocutaneous flap, muscle tissue from the back is used to replace breast tissue. The abdominal muscle is not detached but is still attached to the rectus muscle. The arteries are not relocated. Cognitive Level: Application Text Reference: pp. 1362-1363 Nursing Process: Implementation NCLEX: Physiological Integrity

Which statement by a 32-year-old patient newly diagnosed with stage I breast cancer indicates to the nurse that the goals of therapy are being met? a. "I am not sure how my husband will react when I tell him about this cancer." b. "I am ready to die if that is God's plan for me." c. "I need to know all the options before making a decision about treatment." d. "I will do whatever the doctor thinks is best."

Answer: C Rationale: One goal for the patient with breast cancer is active participation in the decision-making process. The response beginning, "I am not sure how my husband will react" indicates that the goal of satisfaction with the support provided by significant others is still unmet. The response, "I am ready to die if that is God's plan for me" suggests that the patient may not be willing to have treatment. The response, "I will do whatever the doctor thinks is best" indicates that the patient is not participating actively in treatment decisions. Cognitive Level: Application Text Reference: p. 1359 Nursing Process: Evaluation NCLEX: Psychosocial Integrity

A patient returns to the surgical unit following a right modified radical mastectomy with dissection of axillary lymph nodes. An appropriate intervention for the nurse to include in implementing postoperative care for the patient includes a. teaching the patient to use the ordered patient-controlled analgesia (PCA) every 10 minutes for the best pain relief. b. insisting that the patient examine the surgical incision when the dressings are removed. c. posting a sign at the bedside warning against blood pressures or venipunctures in the right arm. d. encouraging the patient to obtain a permanent breast prosthesis as soon as she is discharged from the hospital.

Answer: C Rationale: The patient is at risk for lymphedema and infection if blood pressures or venipuncture are done on the right arm. The patient is taught to use the PCA as needed for pain control rather than at a set time. The nurse allows the patient to examine the incision and participate in care when the patient feels ready. Permanent breast prostheses are usually obtained about 6 weeks after surgery. Cognitive Level: Application Text Reference: pp. 1358-1359 Nursing Process: Implementation NCLEX: Physiological Integrity

Following a modified radical mastectomy, the health care provider recommends chemotherapy even though the lymph nodes were negative for cancer cells. The patient tells the nurse that she does not know what to do about chemotherapy because she has heard that she may not even need chemotherapy and that the side effects are uncomfortable. The nursing diagnosis that best reflects the patient's problem is a. anxiety related to prospect of additional cancer therapy. b. fear related to uncomfortable side effects of chemotherapy. c. decisional conflict related to lack of knowledge about prognosis and treatment options. d. risk for ineffective health maintenance related to reluctance to consider additional treatment.

Answer: C Rationale: The patient's statements indicate that she is having difficulty making a decision about treatment because of a lack of understanding about prognosis and treatment. Although she may have some anxiety and fear, these are not the priorities at this time. The patient expresses concerns about chemotherapy rather than reluctance to consider additional treatment. Cognitive Level: Application Text Reference: p. 1356 Nursing Process: Diagnosis NCLEX: Psychosocial Integrity

A patient states, "I'm sure that I am suffering from premenstrual syndrome (PMS). How can I get my primary health care provider to take this seriously?" What is the nurse's best response? A. "Men are not usually sympathetic to PMS sufferers." B. "You should keep a daily record of the occurrence and severity of your symptoms for three months." C. "Because you feel certain you are right, you should just treat yourself with overthe- counter medications." D. "You are probably right. You should remind your primary health care provider of your symptoms every time you visit."

B. "You should keep a daily record of the occurrence and severity of your symptoms for three months."

A patient with an abnormal mammogram is scheduled for stereotactic core biopsy. Which information will the nurse include when teaching the patient about the procedure? a. "You will need to avoid eating or drinking anything for 6 hours before the procedure." b. "Any discomfort after the biopsy may be treated with mild pain relievers such as aspirin." c. "The core biopsy is evaluated immediately and you will get the results before leaving." d. "Several samples of tissue in the abnormal area will be obtained during the procedure."

Answer: D Rationale: During stereotactic breast biopsy, a biopsy gun is used to remove several core samples in the area of abnormality. The procedure is done using a local anesthetic, so there is no need to be NPO before the procedure. Aspirin should not be used because it will increase bleeding at the site. The biopsy is sent to pathology, and results are not usually available immediately. Cognitive Level: Application Text Reference: p. 1345 Nursing Process: Implementation NCLEX: Health Promotion and Maintenance

While the nurse is obtaining a nursing history from a 52-year-old patient who has found a small lump in her breast, which question is most pertinent? a. "Do you currently smoke cigarettes?" b. "Have you ever had any breast injuries?" c. "Is there any family history of fibrocystic breast changes?" d. "At what age did you start having menstrual periods?"

Answer: D Rationale: Early menarche and late menopause are risk factors for breast cancer because of the prolonged exposure to estrogen that occurs. Cigarette smoking, breast trauma, and fibrocystic breast changes are not associated with increased breast cancer risk. Cognitive Level: Application Text Reference: p. 1349 Nursing Process: Assessment NCLEX: Physiological Integrity

A 34-year-old woman has undergone a modified radical mastectomy for a breast tumor. The pathology report identified the tumor as a stage I, estrogen-receptor-positive adenocarcinoma. The nurse will plan on teaching the patient about a. raloxifene (Evista). b. estradiol (Estrace). c. trastuzumab (Herceptin). d. tamoxifen (Nolvadex).

Answer: D Rationale: Tamoxifen is used for estrogen-dependent breast tumors in premenopausal women. Raloxifene is used to prevent breast cancer, but it is not used post-mastectomy to treat breast cancer. Estradiol will increase the growth of estrogen-dependent tumors. Trastuzumab is used to treat tumors that have the HER-2/neu antigen. Cognitive Level: Application Text Reference: p. 1355 Nursing Process: Planning NCLEX: Physiological Integrity

The nurse provides discharge teaching for a patient who has had a left modified radical mastectomy and axillary lymph node dissection. The nurse determines that teaching has been successful when the patient says, a. "I should keep my left arm supported in a sling when I am up until my incision is healed." b. "I may expose my left arm to the sun for several hours each day to increase circulation and promote healing." c. "I can do whatever exercises and activities I want as long as I do not elevate my left hand above my head." d. "I will continue to exercise my left arm with finger-walking up the wall or combing my hair."

Answer: D Rationale: The patient should continue with arm exercises to regain strength and range of motion. The left arm should be elevated to the level of the heart when the patient is up. Sun exposure is avoided because of the risk of sunburn. The left hand should be elevated at or above heart level to reduce swelling and lymphedema. Cognitive Level: Application Text Reference: pp. 1359-1360 Nursing Process: Evaluation NCLEX: Physiological Integrity

When caring for a 58-year-old patient with persistent menorrhagia, the nurse will plan to monitor the a. estrogen level. b. complete blood count (CBC). c. gonadotropin-releasing hormone (GNRH) level. d. serial human chorionic gonadotropin (hCG) results.

B Because anemia is a likely complication of menorrhagia, the nurse will need to check the CBC. Estrogen and GNRH levels are checked for patients with other problems, such as infertility. Serial hCG levels are monitored in patients who may be pregnant, which is not likely for this patient. DIF: Cognitive Level: Apply (application) REF: 1281 TOP: Nursing Process: Planning MSC:

A 24-year-old patient with pelvic inflammatory disease (PID) is being treated with oral antibiotics as an outpatient. Which instruction will be included in patient teaching? a. Abdominal pain may persist for several weeks. b. Return for a follow-up appointment in 2 to 3 days. c. Instruct a male partner to use a condom during sexual intercourse for the next week. d. Nonsteroidal antiinflammatory drug (NSAID) use may prevent pelvic organ scarring

B The patient is instructed to return for follow-up in 48 to 72 hours. The patient should abstain from intercourse for 3 weeks. Abdominal pain should subside with effective antibiotic therapy. Corticosteroids may help prevent inflammation and scarring, but NSAIDs will not decrease scarring. DIF: Cognitive Level: Apply (application) REF: 1288 TOP: Nursing Process: Implementation MSC:

A 63-year-old woman undergoes an anterior and posterior (A&P) colporrhaphy for repair of a cystocele and rectocele. Which nursing action will be included in the postoperative care plan? a. Encourage a high-fiber diet. b. Perform indwelling catheter care. c. Repack the vagina with gauze daily. d. Teach the patient to insert a pessary.

B The patient will have a retention catheter for several days after surgery to keep the bladder empty and decrease strain on the suture. A pessary will not be needed after the surgery. Vaginal wound packing is not usually used after an A&P repair. A low-residue diet will be ordered after posterior colporrhaphy. DIF: Cognitive Level: Apply (application) REF: 1301 TOP: Nursing Process: Planning MSC:

The nurse is educating a female patient who is experiencing menopausal symptoms about conditions at which the patient will now be at a greater risk. For which conditions will the nurse monitor this patient? Select all that apply. A. Blood clots B. Depression C. Osteoporosis D. Bladder infections E. Endometrial cancer F. Coronary artery disease

B. Depression C. Osteoporosis D. Bladder infections F. Coronary artery disease

A 34-year-old woman who is discussing contraceptive options with the nurse says, "I want to have children, but not for a few years." Which response by the nurse is appropriate? a. "If you do not become pregnant within the next few years, you never will." b. "You may have more difficulty becoming pregnant after about age 35." c. "You have many years of fertility left, so there is no rush to have children." d. "You should plan to stop taking oral contraceptives several years before you want to become pregnant."

B The probability of successfully becoming pregnant decreases after age 35, although some patients may have no difficulty in becoming pregnant. Oral contraceptives do not need to be withdrawn for several years for a woman to become pregnant. Although the patient may be fertile for many years, it would be inaccurate to indicate that there is no concern about fertility as she becomes older. Although the risk for infertility increases after age 35, not all patients have difficulty in conceiving. DIF: Cognitive Level: Apply (application) REF: 1283 TOP: Nursing Process: Implementation MSC:

Which assessment finding in a woman who recently started taking hormone therapy (HT) is most important for the nurse to report to the health care provider? a. Breast tenderness b. Left calf swelling c. Weight gain of 3 lb d. Intermittent spotting

B Unilateral calf swelling may indicate deep vein thrombosis caused by the changes in coagulation associated with HT and would indicate that the HT should be discontinued. Breast tenderness, weight gain, and intermittent spotting are common side effects of HT and do not indicate a need for a change in therapy. DIF: Cognitive Level: Apply (application) REF: 1284 OBJ: Special Questions: Prioritization TOP: Nursing Process: Evaluation MSC:

A 22 year old female patient has come to the clinic for oral contraceptives. The nurse uses the opportunity for patient teaching about BSE. What shoudl the nurse include in patient teaching? A. Women at high risk for breast cancer begin BSE at 3o years old B. Perform BSE monthly on the day of the first pill in each package C. Older women use BSE annually as breast tissue density declines D. Healthy women need to have annual mammograms beginning at age 50

B. The nurse instructs the pt to use the packageing for the oral contraceptives as a convienient reminder to perform BSE monthly to increase adherence

Which of the following female patients should report her results from breast self-examination? A. Denser breast tissue B. Left nipple deviation C. Palpable rib margins D. Different size breasts

B. Unilateral deviation of a nipple and nipple retraction are clinical indicators of breast cancer and should be reported promplty to the HCP. The breasts of most women are slightly different sizes

A patient confides in the nurse that she has started experiencing dyspareunia and asks, "Is there anything I can do about this?" What is the nurse's best response? A. "No, it is part of the aging process." B. "Water-soluble vaginal lubricants may provide relief." C. "You need to be evaluated for a sexually transmitted infection (STI)." D. "You may have vaginal scar tissue that is producing the discomfort."

B. "Water-soluble vaginal lubricants may provide relief."

The term "child sexual abuse" is used for children under what age? A. 12 years B. 14 years C. 18 years D. 21 years

B. 14 years

The nurse has been asked to participate in a healthy living workshop. While teaching about women's health, which of the following guidelines should the nurse provide to the audience? A. "Mammograms are necessary if you have a family history of breast cancer." B. "It's recommended that you get a mammogram each year after you turn 40." C. "If you are not able to perform breast self-examination (BSE), you should go for regular mammograms." D. "You should ensure that your primary care provider performs a breast exam each time you visit."

B. Annual mammograms are recommended after age 40. They are recommended for all women, not solely those with a family history of breast cancer. BSE is not a replacement for mammography and clinical breast examinations are not necessary at each office visit.

A patient reports radiating lower abdominal pain associated with nausea, diarrhea, and fatigue even on the fourth day of menstruation. Which is the best nursing intervention for this patient? A. Advise the patient to use superabsorbent tampons. B. Apply warm compresses to the lower abdomen and back. C. Encourage the patient to perform daily aerobic exercise. D. Advise the patient to eat a hot meal and wear warm clothes.

B. Apply warm compresses to the lower abdomen and back.

A 24-year-old patient who has undergone breast augmentation earlier in the day will be discharged home in the early evening. What instructions should the nurse provide in order to minimize the patient's risk of complications in the immediate recovery period? A. Avoid wearing a bra until postoperative day 3. B. Ask the patient to avoid strenuous exercise during her recovery period. C. Sleep in a semi-Fowler's position until her scheduled follow-up appointment. D. Enlist a friend or family member to perform passive range-of-motion exercises.

B. As with all types of breast surgery, strenuous exercise is contraindicated during the recovery period following breast augmentation. A bra should be worn to prevent dehiscence. Passive range-of-motion exercises should be avoided, and sleeping in a semi-Fowler's position is not necessary

Which is an early clinical sign of cervical cancer? A. Pain B. Asymptomatic C. Heavy, frequent vaginal bleeding D. Dark, foul-smelling vaginal discharge

B. Asymptomatic

A patient is diagnosed with genital herpes infection. What symptoms assessed in the patient should be reported immediately? Select all that apply. A. Diarrhea B. Constipation C. Atonic bladder D. Disorientation or confusion E. Burning sensation in the vesicles

B. Constipation C. Atonic bladder D. Disorientation or confusion

A patient at the clinic reports abdominal bloating, depression, and irritability related to her premenstrual syndrome. What education does the nurse provide? Select all that apply. A. Take diuretics B. Exercise regularly C. Take antidepressants D. Take antianxiety agents E. Increase pork, chicken, and milk intake

B. Exercise regularly E. Increase pork, chicken, and milk intake

A patient with genital herpes reports severe pain and a burning sensation during urination. What are the measures that a nurse can implement to ease this discomfort? Select all that apply. A. Catheterize the patient. B. Give frequent sitz baths to sooth the area. C. Advise the patient to void urine in a warm tub of water or a warm shower. D. Keep the patient on a fluid-only diet and do not allow the patient to move about. E. Use drying agents, such as colloidal oatmeal and aluminum salts, and local anesthetics, such as lidocaine.

B. Give frequent sitz baths to sooth the area. C. Advise the patient to void urine in a warm tub of water or a warm shower. E. Use drying agents, such as colloidal oatmeal and aluminum salts, and local anesthetics, such as lidocaine.

A patient reports dysuria and profuse purulent urethral discharge. During the assessment, the nurse finds that the patient's testicles are swollen and performs a nucleic acid amplification test as prescribed. Which disease does nurse anticipate on the basis of the assessment data? A. Syphilis B. Gonorrhea C. Genital warts D. Chlamydial infection

B. Gonorrhea

The nurse is preparing a patient for external pelvic radiation treatment. What nursing action is essential prior to this treatment? A. Assessing for enteritis B. Having the patient empty the bladder C. Avoiding activities that increase pelvic congestion D. Applying an abdominal binder to prevent distention

B. Having the patient empty the bladder

A patient has tested positive for syphilis. For which other condition should the patient be tested for, based on the positive syphilis test result? A. Tuberculosis B. Human immunodeficiency virus (HIV) C. Genital warts D. Diabetes type 2

B. Human immunodeficiency virus (HIV)

The nurse admits a female patient of childbearing age who reports severe abdominal pain, abnormal uterine bleeding, and constipation. For which diagnostic tests should the nurse anticipate preparing the patient? Select all that apply. A. Myomectomy B. Hysteroscopy C. Hysterectomy D. Pelvic exam and ultrasound E. Complete blood count (CBC) F. Uterine artery embolization (UAE)

B. Hysteroscopy D. Pelvic exam and ultrasound E. Complete blood count (CBC)

The nurse is talking with a couple that are unable to have children. What information should the nurse include? Select all that apply. A. Tobacco and illicit drug use do not cause infertility. B. In certain cases, the cause for infertility may be unidentified. C. The cause for infertility may occur either in the male or the female. D. Assisted reproductive technologies (ARTs) can be used to bear children. E. Presence of infection such as cervicitis may cause infertility even if treated.

B. In certain cases, the cause for infertility may be unidentified. C. The cause for infertility may occur either in the male or the female. D. Assisted reproductive technologies (ARTs) can be used to bear children.

A patient suspected of neurosyphilis underwent cerebrospinal fluid (CSF) analysis. What changes in the CSF does the nurse recognize will indicate neurosyphilis? Select all that apply. A. Increased sugar levels B. Increased white blood cell count C. Increased total proteins D. A positive treponemal antibody test E. Increased creatinine levels

B. Increased white blood cell count C. Increased total proteins D. A positive treponemal antibody tesT

A young woman with polycystic ovary syndrome (PCOS) is on clomiphene therapy. Which is the rationale for use of this medication in this patient? A. Manage insulin resistance B. Induce ovulation C. Manage weight gain D. Reduce cardiovascular risk

B. Induce ovulation

A postmenopausal patient wishes to initiate hormone replacement therapy. What instructions should the nurse give to this patient? Select all that apply. A. Instruct the patient about the maximum effective dose. B. Inform the patient that estrogen and progestin use leads to fewer hip fractures. C. Inform the patient that estrogen and progestin use increases risk of stroke and emboli. D. Instruct the patient that estrogen alone preparations decrease the risk of hip fractures. E. Inform the patient that estrogen alone preparations increase breast and colorectal cancer.

B. Inform the patient that estrogen and progestin use leads to fewer hip fractures. C. Inform the patient that estrogen and progestin use increases risk of stroke and emboli. D. Instruct the patient that estrogen alone preparations decrease the risk of hip fractures.

A patient who does not exhibit signs or symptoms of syphilis receives a positive treponemal antibody test result. On the basis of these data, which stage of syphilis does the nurse suspect? A. Late B. Latent C. Primary D. Secondary

B. Latent

Which condition is characterized as a benign, noncancerous, smooth muscle uterine tumor? A. Fistula B. Leiomyoma C. Endometriosis D. Ectopic pregnancy

B. Leiomyoma

A patient is concerned about the development of cervical cancer because her mother had it. What clinical manifestations does the nurse instruct the patient to immediately report to the primary health care provider? A. Bloating and difficulty in eating B. Leukorrhea and intermenstrual bleeding C. Irregular menstrual periods and amenorrhea D. Abnormal uterine bleeding and bladder discomfort

B. Leukorrhea and intermenstrual bleeding

A patient has a vulvectomy wound, and the nurse is concerned about prevention of fecal contamination of the wound. Which dietary intervention would best aid in the prevention of fecal contamination? A. High-fiber diet B. Low-residue diet C. Increased fat intake D. Increased dairy intake

B. Low-residue diet

A patient seen in the clinic reports a fishy-smelling, thin, white, watery vaginal discharge and is diagnosed with bacterial vaginosis (BV). The patient reports an allergy to sulfa. Which medication does the nurse expect to administer? A. Doxycycline 100 mg PO BID for one week. B. Metronidazole 500 mg PO BID for one week. C. Penicillin G, 2 million units intramuscularly (IM) one time. D. Azithromycin 1 gram by mouth (PO) twice a day (BID) for two weeks.

B. Metronidazole 500 mg PO BID for one week.

A patient presents with primary dysmenorrhea. The nurse is likely to find which clinical presentations during the physical examination? Select all that apply. A. Unilateral pain B. Onset of pain 12-24 hours before menstruation C. Colicky lower abdominal pain radiating to the thighs D. Constant intense pain which lasts for more than two days E. Most severe pain on the first day which subsides within two days

B. Onset of pain 12-24 hours before menstruation C. Colicky lower abdominal pain radiating to the thighs E. Most severe pain on the first day which subsides within two days

When educating a group of teenagers about premenstrual syndrome (PMS), what information should the nurse include? Select all that apply. A. Premenstrual symptoms are similar during each menstrual cycle. B. Premenstrual symptoms can be severe enough to impair interpersonal relationships. C. Premenstrual symptoms occur cyclically in the luteal phase just before menstruation. D. Premenstrual symptoms occur cyclically in the follicular phase just before menstruation. E. Premenstrual symptoms may vary, but breast discomfort is seen in all patients experiencing PMS.

B. Premenstrual symptoms can be severe enough to impair interpersonal relationships. C. Premenstrual symptoms occur cyclically in the luteal phase just before menstruation.

The nurse is working in the emergency department (ED) and caring for a woman admitted for reports of terrible nightmares and nausea accompanied by frequent diarrhea. She has a history of rape during a street mugging. Which of the following would most likely explain the patient's signs and symptoms? A. Contraction of H1N1 B. Rape trauma syndrome C. Experience of an additional rape D. Gastroesophageal reflux disease

B. Rape trauma syndrome

A nurse is screening a person for syphilis. What are the behavior patterns that contribute to an increased risk for developing syphilis? Select all that apply. A. Consumption of polluted water in an area with a high incidence of syphilis B. Sexual intercourse with an infected person C. Sharing of intravenous needles for drugs D. Consumption of food prepared by a person who has syphilis E. Contact with lesions of a person with syphilis

B. Sexual intercourse with an infected person C. Sharing of intravenous needles for drugs E. Contact with lesions of a person with syphilis

A patient with a subdural hematoma is being admitted to the unit from the emergency department (ED). The ED nurse reports that there is a suspicion of sexual abuse. Given this information, what is the most likely reason for a subdural hematoma? A. Bleed from previous injury B. Shoving that resulted in a fall C. Arteriovenous (AV) malformation D. Family history of spontaneous bleeds

B. Shoving that resulted in a fall

A 50-year-old patient is preparing to begin breast cancer treatment with tamoxifen (Nolvadex). Which of the following points should the nurse emphasize when teaching the patient about her new drug regimen? A. "You may find that your medication causes some breast sensitivity." B. "It's important that you let your care provider know about any changes in vision." C. "You'll find that this drug often alleviates some of the symptoms that accompany menopause." D. "It's imperative that you abstain from drinking alcohol after you begin taking tamoxifen."

B. Tamoxifen has the potential to cause cataracts and retinopathy. The drug is likely to exacerbate rather than alleviate perimenopausal symptoms. Breast tenderness is not associated with tamoxifen, and it is not necessary for the patient to abstain from alcohol.

What advice should the nurse give to postmenopausal patients about sexuality? Select all that apply. A. Inform the patient that femininity and libido disappear with menopause. B. Teach the patient that the use of moisturizing soaps and body lotions may help improve dry skin. C. Inform the patient that water-soluble lubricants are effective for atrophic changes in the vagina. D. Inform the patient that cessation of menstruation should be equated with cessation of sexual desire. E. Inform the patient that vaginal atrophic changes are normal after menopause and often are permanent.

B. Teach the patient that the use of moisturizing soaps and body lotions may help improve dry skin. C. Inform the patient that water-soluble lubricants are effective for atrophic changes in the vagina.

A patient has been prescribed tranexamic acid for Heavy Menstrual Bleeding (HMB). What instructions should the nurse give to the patient? Select all that apply. A. The drug cures the HMB. B. The drug only addresses blood loss. C. The drug may cause clots in the blood. D. The drug may also be used for body aches. E. The drug acts by facilitating the action of platelets.

B. The drug only addresses blood loss. C. The drug may cause clots in the blood.

The nurse is providing care to a patient with symptoms of perimenopause. Which findings does the nurse anticipate when performing the health history and physical assessment for this patient? Select all that apply. A. Breast firmness B. Urge incontinence C. Cessation of menses D. Uterine atrophy E. Hot flashes

B. Urge incontinence D. Uterine atrophy E. Hot flashes

A postmenopausal woman states that she has painful sexual intercourse. What advice should the nurse give to this patient? Select all that apply. A. Avoid sexual intercourse. B. Use local hormonal creams. C. Use systemic hormone therapy. D. Ask the male partner to use condoms. E. Use lubricants during sexual intercourse.

B. Use local hormonal creams. C. Use systemic hormone therapy. E. Use lubricants during sexual intercourse.

A 32 year old pt has stage IIB breast cancer that is estrogen receptor positive. What medication should the nurse avoid administering to the patient? A. diethylstilbestrol (DES) B. exemestane (Aromasin) C. trastuzumab (Herceptin) D. doxorubicin (Adriamycin)

B. exemestane (Aromasin) Is an aromatase inhibitor is used in the treatment of postmenopausal women. This drug interferes with the enzyme that helps synthesize endogenous estrogen but does not block the production of ovarian estrogen. Aromatase inhibitors are of little benefir in premenopausal women with breast cancer.

Which information will the nurse include when teaching a patient who has developed a small vesicovaginal fistula 2 weeks into the postpartum period? a. Take stool softeners to prevent fecal contamination of the vagina. b. Limit oral fluid intake to minimize the quantity of urinary drainage. c. Change the perineal pad frequently to prevent perineal skin breakdown. d. Call the health care provider immediately if urine drains from the vagina.

C Because urine will leak from the bladder, the patient should plan to use perineal pads and change them frequently. A high fluid intake is recommended to decrease the risk for urinary tract infections. Drainage of urine from the vagina is expected with vesicovaginal fistulas. Fecal contamination is not a concern with vesicovaginal fistulas. DIF: Cognitive Level: Apply (application) REF: 1301 TOP: Nursing Process: Planning MSC:

A 25-year-old woman who is scheduled for a routine gynecologic examination tells the nurse that she has had intercourse during the last year with several men. The nurse will plan to teach about the reason for a. contraceptive use. b. antibiotic therapy. c. Chlamydia testing. d. pregnancy testing.

C Chlamydia testing is recommended annually for women with multiple sex partners. There is no indication that the patient needs teaching about contraceptives, pregnancy testing, or antibiotic therapy. DIF: Cognitive Level: Apply (application) REF: 1287 TOP: Nursing Process: Planning MSC:

A 49-year-old woman is considering the use of combined estrogen-progesterone hormone replacement therapy (HT) during menopause. Which information will the nurse include during their discussion? a. Use of estrogen-containing vaginal creams provides most of the same benefits as oral HT. b. Increased incidence of colon cancer in women taking HT requires more frequent colonoscopy. c. HT decreases osteoporosis risk and increases the risk for cardiovascular disease and breast cancer. d. Use of HT for up to 10 years to prevent symptoms such as hot flashes is generally considered safe.

C Data from the Women's Health Initiative indicate an increased risk for cardiovascular disease and breast cancer in women taking combination HT but a decrease in hip fractures. Vaginal creams decrease symptoms related to vaginal atrophy and dryness, but they do not offer the other benefits of HT, such as decreased hot flashes. Most women who use HT are placed on short-term treatment and are not treated for up to 10 years. The incidence of colon cancer decreases in women taking HRT. DIF: Cognitive Level: Apply (application) REF: 1284 TOP: Nursing Process: Implementation MSC:

A 27-year-old patient tells the nurse that she would like a prescription for oral contraceptives to control her premenstrual dysphoric disorder (PMD-D) symptoms. Which patient information is most important to communicate to the health care provider? a. Bilateral breast tenderness b. Frequent abdominal bloating c. History of migraine headaches d. Previous spontaneous abortion

C Oral contraceptives are contraindicated in patients with a history of migraine headaches. The other patient information would not prevent the patient from receiving oral contraceptives. DIF: Cognitive Level: Apply (application) REF: 1280 TOP: Nursing Process: Assessment MSC:

An 18-year-old requests a prescription for birth control pills to control severe abdominal cramping and headaches during her menstrual periods. Which should the nurse take first? a. Determine whether the patient is sexually active. b. Teach about the side effects of oral contraceptives. c. Take a personal and family health history from the patient. d. Suggest nonsteroidal antiinflammatory drugs (NSAIDs) for relief.

C Oral contraceptives may be appropriate to control this patient's symptoms, but the patient's health history may indicate contraindications to oral contraceptive use. Because the patient is requesting contraceptives for management of dysmenorrhea, whether she is sexually active is irrelevant. Because the patient is asking for birth control pills, responding that she should try NSAIDs is nontherapeutic. The patient does not need teaching about oral contraceptive side effects at this time. DIF: Cognitive Level: Apply (application) REF: 1280 OBJ: Special Questions: Prioritization TOP: Nursing Process: Implementation MSC:

A 47-year-old woman asks whether she is going into menopause if she has not had a menstrual period for 3 months. The best response by the nurse is which of the following? a. "Have you thought about using hormone replacement therapy?" b. "Most women feel a little depressed about entering menopause." c. "What was your menstrual pattern before your periods stopped?" d. "Since you are in your mid-40s, it is likely that you are menopausal."

C The initial response by the nurse should be to assess the patient's baseline menstrual pattern. Although many women do enter menopause in the mid-40s, more information about this patient is needed before telling her that it is likely she is menopausal. Although hormone therapy (HT) may be prescribed, further assessment of the patient is needed before discussing therapies for menopause. Because the response to menopause is very individual, the nurse should not assume that the patient is experiencing any adverse emotional reactions. DIF: Cognitive Level: Apply (application) REF: 1283-1285 TOP: Nursing Process: Implementation MSC:

The nurse has just received change-of-shift report about the following four patients. Which patient should be assessed first? a. A patient with a cervical radium implant in place who is crying in her room b. A patient who is complaining of 5/10 pain after an abdominal hysterectomy c. A patient with a possible ectopic pregnancy who is complaining of shoulder pain d. A patient in the fifteenth week of gestation who has uterine cramping and spotting

C The patient with the ectopic pregnancy has symptoms consistent with rupture and needs immediate assessment for signs of hemorrhage and possible transfer to surgery. The other patients should also be assessed as quickly as possible but do not have symptoms of life-threatening complications. DIF: Cognitive Level: Analyze (analysis) REF: 1282 OBJ: Special Questions: Prioritization; Multiple Patients TOP: Nursing Process: Assessment MSC:

A patient has undergone induced abortion. The nurse is evaluating the patient after teaching about necessary interventions to avoid complications. Which statement made by the patient indicates effective learning? A. "I should perform vaginal douching." B. "I should avoid oral contraceptive pills." C. "I should avoid intercourse for two weeks." D. "I should perform exercise for one hour every day."

C. "I should avoid intercourse for two weeks."

The nurse is educating a patient about care after a hysterectomy. Which statement indicates to the nurse that the patient needs further education? A. "I will avoid dancing for several months." B. "I will avoid swimming for 8 weeks." C. "I will avoid sexual intercourse for 2 weeks." D. "I will avoid heavy lifting for 2 months."

C. "I will avoid sexual intercourse for 2 weeks."

A nurse in a family planning clinic is completing the admission intake on a woman who has decided to get an abortion for an unwanted pregnancy. The patient tells the nurse that she is struggling with her decision because she is concerned that God will punish her for this decision and that she will not be able to reconcile this decision as she continues with life. What is the most appropriate response from the nurse? A. "God is forgiving; forgiveness will be given and your spirit will heal over time." B. "I understand why you feel that way, but this is the best decision at this point." C. "Would you like to speak with someone about these concerns before the procedure?" D. "Spirituality concerns should have been considered before scheduling of the appointment."

C. "Would you like to speak with someone about these concerns before the procedure?"

Which instruction should the nurse provide regarding clomiphene, prescribed for a patient with ovarian cysts? A. "You should contact the primary health care provider if you gain weight." B. "You should contact the primary health care provider if you get irregular menses." C. "You should contact the primary health care provider if lower abdominal pain occurs." D. "You should contact the primary health care provider if you find abnormal hair on your body."

C. "You should contact the primary health care provider if lower abdominal pain occurs."

The nurse is educating a postmenopausal woman who is not receiving hormone replacement therapy (HRT) about calcium supplementation. How many milligrams (mg) per day is the recommended dose? A. 500 mg B. 1,000 mg C. 1,500 mg D. 2,000 mg

C. 1,500 mg

Spontaneous abortion is the term used to describe the natural loss of a pregnancy prior to how many weeks of gestation? A. 10 B. 12 C. 20 D. 25

C. 20

Which patient will the nurse educate about oral contraceptives? A. A 17-year-old patient with vaginal discharge that smells fishy B. A 34-year-old patient with implantation of a fetus in her ovaries C. A 46-year-old patient with irregular menses for the past two years D. A 21-year-old patient with unknown abdominal bleeding and pain

C. A 46-year-old patient with irregular menses for the past two years

A pregnant female is experiencing amenorrhea, morning sickness, and breast tenderness. In the ninth week after her last menstrual period she is rushed to the hospital with severe left shoulder pain, blood pressure of 90/60 mm Hg, and heart rate of 112 beats/minute. What is the best diagnostic test to determine the cause of her problem? A. Serum hemoglobin B. 12-lead electrocardiogram C. A transvaginal ultrasound D. Serial β-human chorionic gonadotropin levels

C. A transvaginal ultrasound

A woman infected with gonorrhea has a vaginal birth. What precautions should the nurse follow to protect the baby from complications of gonorrhea? Select all that apply. A. Administer paracetamol drops to the baby. B. Administer amoxicillin solution to the baby. C. Administer silver nitrate aqueous solution to the baby. D. Administer erythromycin ophthalmic ointment to the baby. E. Administer an intramuscular dose of ceftriaxone to the baby.

C. Administer silver nitrate aqueous solution to the baby. D. Administer erythromycin ophthalmic ointment to the baby.

A nurse is teaching a health promotion workshop to a group of women in their 40s and 50s. Which of the following points about nipple discharge should the nurse teach to participants? A. Inappropriate lactation necessitates breast biopsy. B. Nipple discharge of any type is considered a precursor to cancer. C. Unexpected nipple discharge of any type warrants medical follow-up. D. Galactorrhea is a normal age-related change and a frequent perimenopausal symptom.

C. Although most cases of nipple discharge are not related to malignancy, further assessment is indicated. Galactorrhea is not considered a normal age-related change, nor is it a common perimenopausal symptom.

The nurse is educating a patient regarding a new medication regimen that may lead to amenorrhea. Which medication will the nurse discuss with this patient? A. Clonidine B. Raloxifene C. Amitriptyline D. Norethindrone

C. Amitriptyline

The nurse is caring for a patient diagnosed with breast cancer who just underwent an axillary lymph node dissection. Which of the following interventions would the nurse use to decrease the lymphedema? A. Keep affected arm flat at the patient's side. B. Apply an elastic bandage on the affected arm. C. Assess blood pressure on unaffected arm only. D. Restrict exercise of the affected arm for 1 week.

C. Blood pressure readings, venipunctures, and injections should not be done on the affected arm. Elastic bandages should not be used in the early postoperative period because they inhibit collateral lymph drainage. The affected arm should be elevated above the heart, and isometric exercises are recommended to reduce fluid volume in the arm.

A patient is diagnosed with cancer that has spread beyond the cervix. For which type of therapy should the nurse prepare the patient? A. Cautery B. Cryosurgery C. Cisplatin-based chemotherapy D. Tamoxifen and progesterone therapy

C. Cisplatin-based chemotherapy

Because of the risks associated with hormone therapy (HT), a female patient does not want to take HT and asks the nurse how she can tolerate the perimenopausal symptoms of hot flashes and sweating at night. What is the best response by the nurse? A. Increase warmth to avoid chills. B. Maintain good nutrition to avoid osteoporosis. C. Decrease heat production and increase heat loss. D. Take vitamin B complex and use vaginal lubrication.

C. Decrease heat production and increase heat loss.

A patient suspects she is having symptoms related to menopause. Which diagnostic study will the nurse tell the patient is used to confirm the diagnosis of menopause? A. Ultrasound B. Endometrial biopsy C. Follicle-stimulating hormone (FSH) D. Human chorionic gonadotropin (HCG)

C. Follicle-stimulating hormone (FSH)

A female patient reports the recent appearance of itchy lesions on her vulva, some of which have burst recently. What problem should the nurse first suspect related to the patient's description of her problem? A. Gonorrhea B. Chlamydia C. Genital herpes D. Human immunodeficiency virus (HIV)

C. Genital herpes

A patient arrives at the clinic to have a Papanicolaou (Pap) test done. What information should the nurse give to this patient? Select all that apply. A. Get this test done annually until the age of 65 years. B. Get this test done three times a year until the age of 65 years. C. Get this test done once in three years until the age of 65 years. D. This test is not required for women who have undergone hysterectomy. E. After 65 years, this test is not required if two or three consecutive negative Pap tests are received in two years.

C. Get this test done once in three years until the age of 65 years. D. This test is not required for women who have undergone hysterectomy. E. After 65 years, this test is not required if two or three consecutive negative Pap tests are received in two years.

A 29-year-old primiparous patient has a 3-week-old infant whom she is breastfeeding. The woman has sought care because of recent breast tenderness, redness, and fever. Which of the following teaching points should the nurse prioritize when following up her care? A. Encourage patient to continue breastfeeding her infant. B. Refer patient for a mammogram as quickly as possible. C. Ensure patient adheres to her prescribed antibiotic regimen. D. Teach patient to use warm compresses and educate her about self-limiting nature of illness.

C. Mastitis normally requires antibiotic therapy, the success of which is often dependent on close adherence to the prescribed regimen. Breastfeeding should indeed be continued if possible, but effective treatment of her infection would be the immediate priority. Mastitis is not necessarily self limiting and mammography is not normally indicated.

Primary dysmenorrhea is most commonly caused by overproduction of which hormone? A. Estrogen B. Progesterone C. Prostaglandin D. Human chorionic gonadotropin (HCG)

C. Prostaglandin

A female patient reports severe back pain and abdominal cramping during her menstrual cycle, and the nurse educates the patient about taking nonsteroidal antiinflammatory drugs (NSAIDS) and using heat to relieve her discomfort. What explanation does the nurse give for how NSAIDs offer relief from this type of pain? A. Progesterone stimulates an increase in the uterine pain receptors. B. Increased estrogen results in a decrease in prostaglandin production. C. Prostaglandin production increases due to estrogen and progesterone. D. Prostaglandin production is below normal, triggering cramping in the uterus.

C. Prostaglandin production increases due to estrogen and progesterone.

The nurse is caring for a patient who is postoperative after undergoing a vulvectomy. What is the priority nursing action to prevent wound contamination? A. Administering analgesics B. Using indwelling catheter C. Providing low residue diet D. Administering anticoagulants

C. Providing low residue diet

A patient is scheduled to have a surgical procedure involving dissection of lymph nodes in the pelvis. About what surgical procedure should the nurse educate the patient? A. Vulvectomy B. Vaginectomy C. Radical hysterectomy D. Vaginal hysterectomy

C. Radical hysterectomy

A patient that left a tampon in for several days is suspected of having developed toxic shock syndrome. What does the nurse recognize are clinical manifestations of this syndrome? A. Hypotension B. Hemorrhage C. Rapid onset of high fever D. Sudden sharp abdominal pain

C. Rapid onset of high fever

A patient with primary dysmenorrhea inquires as to what interventions may be used in order to relieve the discomfort. What is the best response by the nurse? Select all that apply. A. Steroids B. Hysterectomy C. Regular exercises D. Nonsteroidal antiinflammatory drugs E. Application of heat to lower abdomen and back

C. Regular exercises D. Nonsteroidal antiinflammatory drugs E. Application of heat to lower abdomen and back

A patient is scheduled to have a surgical procedure to remove a fallopian tube after an ectopic pregnancy. What procedure should the nurse ensure is scheduled on the consent? A. Vulvectomy B. Hysterectomy C. Salpingectomy D. Oophorectomy

C. Salpingectomy

The nurse is providing care to a patient who has been admitted for surgical removal of the fallopian tubes. How does the nurse document this surgical procedure in the patient's medical record? A. Hysterectomy B. Myomectomy C. Salpingectomy D. Oophorectomy

C. Salpingectomy

A college student accompanied by a roommate presents to the emergency department (ED) to seek treatment for worsening depression. What further assessment from the nurse should be considered as a cause of the depression? Select all that apply. A. Conflicts with roommate B. Week-long sinus infection C. Sexual assault on campus D. Difficulty in college courses E. Family history of depression

C. Sexual assault on campus D. Difficulty in college courses E. Family history of depression

A patient with excessive bleeding for more than a week is on tranexamic acid therapy. Which outcome in the patient does the nurse recognize indicates effective treatment? A. The patient will not experience anxiety. B. The patient will not experience migraine headache. C. The patient will not experience heavy menstrual bleeding. D. The patient will not experience bacterial and fungal infection. The nurse is reviewing the medical history of few patients in the unit.

C. The patient will not experience heavy menstrual bleeding.

A patient with a chlamydial infection has completed the course of treatment. What advice should the nurse provide to this patient to avoid reinfection? Select all that apply. A. Avoid heavy exercise. B. Use oral contraceptive pills. C. Use condoms in the future during sexual activity. D. Have the sexual partner or partners screened and treated. E. Avoid sexual intercourse for seven days after finishing treatment.

C. Use condoms in the future during sexual activity. D. Have the sexual partner or partners screened and treated. E. Avoid sexual intercourse for seven days after finishing treatment.

The nurse is educating a patient with a sexually transmitted infection about hygiene tips. What should the nurse include when discussing this with the patient? Select all that apply. A. Perform regular douching B. Avoid using public toilets C. Use cotton underwear D. Frequently wash hands and regularly bathe E. Avoid any physical contact with strangers

C. Use cotton underwear D. Frequently wash hands and regularly bathe

A patient with Heavy Menstrual Bleeding (HMB) is concerned about her condition. When explaining HMB to this patient to alleviate her anxiety, what information should the nurse include? Select all that apply. A. Ovulatory cycles are commonly associated with HMB. B. Unopposed progesterone action on endometrium results in HMB. C. Uterine fibroid and endometrial polyps are commonly associated with HMB. D. Menstrual bleeding either lasts more than seven days, amounts to more than 80 mL, or both. E. Menstrual bleeding either lasts for more than seven days or amounts to more than 80 mL, but never both.

C. Uterine fibroid and endometrial polyps are commonly associated with HMB. D. Menstrual bleeding either lasts more than seven days, amounts to more than 80 mL, or both.

A 31-year-old patient has just been instructed in the treatment for a Chlamydia trachomatis vaginal infection. Which patient statement indicates that the nurse's teaching has been effective? a. "I can purchase an over-the-counter medication to treat this infection." b. "The symptoms are due to the overgrowth of normal vaginal bacteria." c. "The medication will need to be inserted once daily with an applicator." d. "Both my partner and I will need to take the medication for a full week."

D Chlamydia is a sexually transmitted bacterial infection that requires treatment of both partners with antibiotics for 7 days. The other statements are true for the treatment of Candida albicans infection. DIF: Cognitive Level: Apply (application) REF: 1287 TOP: Nursing Process: Evaluation MSC:

A 19-year-old has been diagnosed with primary dysmenorrhea. How will the nurse suggest that the patient prevent discomfort? a. Avoid aerobic exercise during her menstrual period. b. Use cold packs on the abdomen and back for pain relief. c. Talk with her health care provider about beginning antidepressant therapy. d. Take nonsteroidal antiinflammatory drugs (NSAIDs) when her period starts.

D NSAIDs should be started as soon as the menstrual period begins and taken at regular intervals during the usual time frame in which pain occurs. Aerobic exercise may help reduce symptoms. Heat therapy, such as warm packs, is recommended for relief of pain. Antidepressant therapy is not a typical treatment for dysmenorrhea. DIF: Cognitive Level: Apply (application) REF: 1280 TOP: Nursing Process: Implementation MSC:

To prevent pregnancy in a patient who has been sexually assaulted, the nurse in the emergency department will plan to teach the patient about the use of a. mifepristone (RU-486). b. dilation and evacuation. c. methotrexate with misoprostol. d. levonorgestrel (Plan-B One-Step).

D Plan B One-Step reduces the risk of pregnancy when taken within 72 hours of intercourse. The other methods are used for therapeutic abortion, but not for pregnancy prevention after unprotected intercourse. DIF: Cognitive Level: Understand (comprehension) REF: 1303 TOP: Nursing Process: Planning MSC:

When caring for a patient who has a radium implant for treatment of cancer of the cervix, the nurse will a. assist the patient to ambulate every 2 to 3 hours. b. use gloves and gown when changing the patient's bed. c. flush the toilet several times right after the patient voids. d. encourage the patient to discuss needs or concerns by telephone.

D The nurse should spend minimal time in the patient's room to avoid exposure to radiation. The patient and nurse can have longer conversations by telephone between the patient room and nursing station. To prevent displacement of the implant, absolute bed rest is required. Wearing of gloves and gown when changing linens, and flushing the toilet several times are not necessary because the isotope is confined to the implant. DIF: Cognitive Level: Apply (application) REF: 1297 | 1299 TOP: Nursing Process: Implementation MSC:

The nurse in the women's health clinic has four patients who are waiting to be seen. Which patient should the nurse see first? a. 22-year-old with persistent red-brown vaginal drainage 3 days after having balloon thermotherapy b. 42-year-old with secondary amenorrhea who says that her last menstrual cycle was 3 months ago c. 35-year-old with heavy spotting after having a progestin-containing IUD (Mirena) inserted a month ago d. 19-year-old with menorrhagia who has been using superabsorbent tampons and has fever with weakness

D The patient's history and clinical manifestations suggest possible toxic shock syndrome, which will require rapid intervention. The symptoms for the other patients are consistent with their diagnoses and do not indicate life-threatening complications. DIF: Cognitive Level: Analyze (analysis) REF: 1282 OBJ: Special Questions: Prioritization; Multiple Patients TOP: Nursing Process: Assessment MSC:

The nurse is educating a patient with abnormal uterine bleeding. Which statement made by the patient indicates the needs for further instruction? A. "I should regularly change my tampon." B. "I should record the degree of pad saturation daily." C. "I should get my blood pressure checked regularly." D. "I should apply heat on my lower back and abdomen."

D. "I should apply heat on my lower back and abdomen."

A nurse is educating a patient with a diagnosis of menopause who is not receiving hormone replacement therapy about strategies to prevent or reduce the risk of osteoporosis. Which patient statement is most important for the nurse to determine that the education is understood? A. "Exercise will keep me and my bones strong." B. "I will go outside to make sure I get enough vitamin D." C. "Good nutrition is important to reduce my risk of osteoporosis." D. "I should have an intake of at least 1,500 mg of calcium every day."

D. "I should have an intake of at least 1,500 mg of calcium every day."

A patient with premenstrual syndrome (PMS) states, "My PMS symptoms get so bad sometimes. I feel like my boyfriend thinks it is all in my head." What is the most therapeutic response from the nurse? A. "PMS symptoms are different for everyone." B. "I can totally relate. You shouldn't worry about what your boyfriend thinks." C. "There are a lot of different medications you can take to treat your symptoms." D. "PMS symptoms are physiological. Let's talk about different ways you can manage your symptoms."

D. "PMS symptoms are physiological. Let's talk about different ways you can manage your symptoms."

A patient who is postmenopausal comes to the clinic reporting unexpected vaginal bleeding. What should the nurse tell the patient about diagnosing the cause of this bleeding? A. "It is probably only the end of menopause." B. "You will need a hysterectomy to treat this bleeding." C. "You will need a Pap smear to see if you have endometrial cancer." D. "You will need an endometrial biopsy to determine the cause of bleeding."

D. "You will need an endometrial biopsy to determine the cause of bleeding."

A patient reports the absence of menstruation, abnormal growth of hair of the body, weight gain, and infertility problems. What does the nurse infer from these clinical manifestations? A. Leiomyomas B. Cervical polyps C. Cervical cancer D. Polycystic ovary syndrome

D. Polycystic ovary syndrome

An older adult female patient taking hormone replacement therapy (HRT) is admitted to the unit with unexplained weight loss and ascites of her abdomen. The health care provider drains 500 mL of ascites fluid from the abdominal cavity using ultrasound-guided aspiration. What assessment questions should the nurse include in the documentation? Select all that apply. A. Has the patient ever had gallstones? B. When was the patient's last physical exam? C. Did the patient have her appendix removed? D. At what age did the patient begin menstruation? E. What type of diet does the patient follow at home? F. Does the patient have a family history of breast cancer?

D. At what age did the patient begin menstruation? E. What type of diet does the patient follow at home? F. Does the patient have a family history of breast cancer?

A patient has a history of familial sexual abuse and was the victim of a sexual assault three weeks prior. This week, the patient misses an appointment. Which of the following is the priority nursing intervention? A. Ask the health care provider how to handle this situation. B. Call the patient during break to offer verbal support by phone. C. Ensure the patient is scheduled for an appointment the following week. D. Call the patient expediently to assess why they missed the appointment.

D. Call the patient expediently to assess why they missed the appointment.

Upon examination of a male patient, the nurse notes profuse, purulent urethral discharge and the patient states that he has dysuria and "painful testicles." Based on these findings, which medication does the nurse anticipate the primary health care provider will prescribe? A. Acyclovir B. Penicillin G C. Podofilox D. Ceftriaxone

D. Ceftriaxone

A known sex worker is brought into the emergency department (ED). She had been working and arrived with facial trauma including a laceration to the lip and eyebrow, and swelling of the right eye. What is the most appropriate nursing action? A. Cleanse and treat the facial injuries. B. Obtain fresh clothing and discard clothing she is wearing. C. Call the patient's boyfriend to transport her upon discharge. D. Consult sexual assault nurse examiner (SANE) for rape examination.

D. Consult sexual assault nurse examiner (SANE) for rape examination.

The nurse is preparing to provide teaching to a patient who has been diagnosed with endometrial cancer. Which information does the nurse include in the teaching plan? A. Metastasis is very rare. B. Pain occurs at the early stages of the disease. C. Endometrial cancer can be cured with partial hysterectomy. D. During distant metastasis, the lungs, bones, and brain are involved.

D. During distant metastasis, the lungs, bones, and brain are involved.

A patient had a surgical repair of a fistula. What education should the nurse provide to the patient to avoid complications related to the surgical procedure? A. Douche daily to prevent postoperative infection B. Remove and cleanse her pessary on a daily basis C. Resume normal activity to prevent adhesion formation D. Ensure that she does not place stress on the repaired area

D. Ensure that she does not place stress on the repaired area

The nurse is providing care to a female patient that experienced a sexual assault. What is the priority nursing action for this patient? A. Administering a pregnancy test B. Close monitoring of the patient's vital signs C. Ensuring the patient is left alone when possible D. Informing the patient about possible financial support

D. Informing the patient about possible financial support

A patient is concerned regarding some of the symptoms being experienced after menopause. What information given by the nurse would explain this process? A. This condition will cause libido to disappear. B. This condition will cause femininity to disappear. C. It is a condition in which increased menstrual bleeding occurs. D. It is a condition in which atrophic changes in the reproductive tissues are observed.

D. It is a condition in which atrophic changes in the reproductive tissues are observed.

A patient is diagnosed with an unruptured ectopic pregnancy. What medication does the nurse anticipate educating the patient about to treat this condition? A. Tamoxifen B. Leucovorin C. Gemcitabine D. Methotrexate

D. Methotrexate

A 51-year-old woman has recently had a unilateral, right total mastectomy and axillary node dissection for the treatment of breast cancer. Which of the following interventions should the nurse include in the patient's care? A. Immobilize the patient's right arm until postoperative day 3. B. Maintain the patient's right arm in a dependent position when at rest. C. Administer diuretics prophylactically for the prevention of lymphedema. D. Promote gradually increasing mobility as soon as possible following surgery.

D. Mobility should be encouraged beginning in postanesthetic recovery and increased gradually throughout the patient's recovery. Immobilization is counterproductive to recovery and the limb should never be in a dependent position. Diuretics are not used to prevent lymphedema but may be used in active treatment of the problem.

The nurse is assessing a patient suspected of having syphilis. What medication should the nurse prepare to administer? A. Podophyllin resin B. Acyclovir C. Azithromycin D. Penicillin G benzathine

D. Penicillin G benzathine

The nurse is caring for a postoperative ovarian cancer patient the day after she had a tumor debulking. The patient is crying and refusing to allow her husband of 25 years to visit. What is the most likely reason for the patient's refusal to allow her husband to visit? A. Experiencing too much abdominal pain for visitors B. Emotional and overly tired due to a noisy care environment C. Concerned about privacy when the nurse performs the dressing change D. Perceived perception of the husband's feelings about this type of surgery

D. Perceived perception of the husband's feelings about this type of surgery

A patient has a diagnosis of endometriosis after having symptoms for several years and not knowing what the problem was. What nursing action is appropriate for this patient? A. Reassure the patient that it is not a life-threatening condition, but a hysterectomy will be necessary. B. Explain to the patient that weight management and exercises are important components of treatment. C. Inform the patient that it is a very complicated disease, and she should be prepared for vigorous medical and surgical management. D. Reassure the patient that it is not a life-threatening condition, but she should be prepared for a conservative and progressive treatment approach.

D. Reassure the patient that it is not a life-threatening condition, but she should be prepared for a conservative and progressive treatment approach.

The nurse is assessing a patient with polycystic ovary syndrome who has excessive body hair. Which medication does the nurse anticipate to be most beneficial for the patient? A. Orlistat B. Metformin C. Clomiphene D. Spironolactone

D. Spironolactone

The nurse is caring for a patient with polycystic ovarian syndrome. What nursing interventions are appropriate for this patient? A. Advise absolute bed rest. B. Instruct to take diet rich in carbohydrates and fat. C. Explain the advantages of early surgical intervention. D. Suggest use of a depilating agent or electrolysis to remove unwanted hair.

D. Suggest use of a depilating agent or electrolysis to remove unwanted hair.

A patient is diagnosed with endometriosis after having pain for many years. After having 2 children via in vitro, she is seeking a cure for this problem. What treatment should the nurse educate the patient regarding? A. Danazol B. Leuprolide C. Nonsteroidal antiinflammatory drugs D. Surgical removal of endometrial implants

D. Surgical removal of endometrial implants


Ensembles d'études connexes

Chapter 8: Business Plans: Seeing Audiences and Your Business Clearly

View Set

SKELETAL MUSCLE - STRUCTURE AND FUNCTION

View Set

Level G, Unit 4: Completing the Sentence, Level G. Unit 4:

View Set

Chapter 30: Assessment and Management of Patients with Vascular Disorders and Problems of Peripheral Circulation NCLEX

View Set

Chapter 2: Collecting Subjective Data: The Interview and Health History

View Set

Fundamentals Chapter 5 PrepU Questions

View Set